You are on page 1of 730
OBJECTIVE | ELECTRICAL TECHNOLOGY For the Students of U.P.S.C. (Engg. Services); L.A.S. (Engg. Group); B.Sc. Engg.; Diploma and Other Competitive Courses. (Over 2800 Objective Questions with Hints) V.K. MEHTA ROHIT MEHTA {ip S. CHAND & COMPANY LTD. (AN ISO 9001 : 2000 COMPANY) RAM NAGAR, NEW DELHI-110 055 S. CHAND & COMPANY LTD. = (An iSO 9001 : 2000 Company) _ Head Office : 7361, RAM NAGAR, NEW DELHI - 110055 Phones : 23672080-81-82, 9899107446, 991 1310888; Fax: 91-11-23677446 Shop at: schandgroup.com; E-mail: schand@vsni.com Branches : * Ist Floor, Heritage, Near Gujarat Vichyapeeth, Ashram Road, Ahmedabad-380014,Ph.27541965, 27542369, chmedabod@schandgroup.com © No.6, Ahuja Chambers, Ist Cross, Kumara Krupa Road, Bangalore-56000 1, Ph: 22268048, 22354008, bangalore@schandgroup. com © 238-A M.P. Nogar. Zone |, Bhopal - 452011. Ph : 4274723. bhopal@schandgroup.com # 182, Anna Salol, Chennai-600002. Ph: 28460026, chennai@schandgroup.com # S.C.0. 6,7 &8,Sector 9D; Chandigarh- 160017, Ph-2749376,2749377, chandigarh@schandgroup.com ® Ist Floor, Bhartia Tower, Badambadi. Cuttock-753 009, Ph-2332580; 2332581, cuttack@schandgroup.com. © Ist Floor. 52-A, Rajpur Road, Dehradun-248 001. Ph : 2740889, 2740861, dehracun@schandgroup.com Pan Bazar, Guwahati-78 1 001.Ph:2514155, guwohati@schandgroup.com ® Sultan Bazar, Hydetabad-500 195, Ph :24651 136,24744815, nyderadad@schandgroup.com * MaiHiran Gate, Jalandhar- 144008 . Ph.2401630, jolandhar@schandgroup.com *@ A-14 Janta Store Shopping Complex, University Marg, Bapu Nagar, Jaipur - 302 015, Phone :2719126, joipur@schandgroup.com * 613-7.M.G. Road, Emokulam,Kochi-682035. Ph: 2381740, cochin@schandgroup.com # 285/4, Bipin Bihari GangullStreet, Kolkata-700012. Ph: 22367459, 22373914, kolkata@schandgroup.com * MahabeerMarket,25 Gwynne Road, Aminabad, Lucknow-226018, Ph: 2626801,2284815, lucknow@schandgroup.com @ Blackie House, 103/5, WalchandHirachandMarg , Opp. G.?.0., Mumbai-400001 Phy:2269088 1,226 10885, mumbai@schandgroup.com * Kamal Bag, Mode! Mill Chowk, Umrer Road, Nagpur-440 032 Ph: 2723901, 2777666 nagpur@schandgoup.com # 104.Citicentre Ashok. Govind Mitra Road, Patna-800004. Ph: 230089,2302100, patna@schandgroup.com © 2002, V.K. Mehta & Rohit Mehta All rights reserved. No part of this publication may be reproduced, stored in a retrieval system or transmitted, in any form or by any means, electronic, mechanical, photocopying, recording or otherwise, without the prior permission of the Publishers. First Edition 2002 Reprint with Corrections 2004 Revised Edition 2005 Revised Edition 2007 ISBN : 81-219-2097-3 Code : 10 233 PRINTED IN INDIA By Rajendra Ravinara Printers (Pvt.) itd., 7361, Ram Nagar, New Delhi-1 10 055 and published by S$. Chand & Company Ltd. 7361. Ram Nagar. New Deihi-1 10 055 CONTENTS Part-I : Basic Electrical Engineering 1, Basic Concepts Chapter Overview Short Answer Questions Objective Questions Answers to Objective Questions Hints to Selected Objective Questions 2._D.C. Circuits Chapter Overview Short Answer Questions Objective Questions ‘Answers to Objective Questions Hints to Selected Objective Questions 3.__Network Theorems Chapter Overview Short Answer Questions Objective Questions Answers to Objective Questions Hints to Selected Objective Questions 4. Electrical Work, Power and Energy Chapter Overview ‘Short Answer Questions Objective Questions Answers to Objective Questions Hints to Selected Objective Questions 5. Electrostatics Chapter Overview Short Answer Questions Objective Questions Answers to Objective Questions Hints to Selected Objective Questions 6. Capacitance Chapter Overview Short Answer Questions Objective Questions Answers to Objective Questions Hints to Selected Objective Questions ” 7 10. i. 12. (vi) Magnetism and Electromagnetism Chapter Overview Short Answer Questions Objective Questions Answers to Objective Questions Hints to Selected Objective Questions |. Magnetic Circuits Chapter Overview Short Answer Questions Objective Questions Answers to Objective Questions Hints to Selected Objective Questions Electromagnetic Induction Chapter Overview Short Answer Questions Objective Questions Answers to Objective Questions Hints to Selected Objective Questions Chemical Effects of Electric Current Chapter Overview Short Answer Questions Objective Questions Answers to Objective Questions Hints to Selected Objective Questions Alternating Current Chapter Overview Short Answer Questions Objective Questions Answers to Objective Questions Hints to Selected Objective Questions Series A.C. Circuits Chapter Overview Short Answer Questions Objective Questions Answers to Objective Questions Hints to Selected Objective Questions Phasor Algebra Chapter Overview Short Answer Questions Objective Questions Answers to Objective Questions Hints to Selected Objective Questions ee) 246 —271 = 246 246 262 267 302 — 329 302 312 317 325 326 15. 16. 17, 18. 19. 20, (vii) Parallel A.C. Circuits Chapter Overview Short Answer Questions Objective Questions Answers to Objective Questions Hints to Selected Objective Questions Three Phase Circuits Chapter Overview Short Answer Questions Objective Questions Answers to Objective Questions Hints to Selected Objective Questions Electrical Measuring Instruments Chapter Overview Short Answer Questions Objective Questions Answers to Objective Questions Hints to Selected Objective Questions Part-II : Electrical Machines D.C. Generators Chapter Overview ‘Short Answer Questions Objective Questions Answers to Objective Questions Hints to Selected Objective Questions D.C. Motors Chapter Overview Short Answer Questions Objective Questions Answers to Objective Questions Hints to Selected Objective Questions ‘Transformers Chapter Overview Short Answer Questions Objective Questions Answers to Objective Questions Hints to Selected Objective Questions Three Phase Induction Motors Chapter Overview Short Answer Questions Objective Questions Answers to Objective Questions Hints to Selected Objective Questions 353 — 386 353 367 374 381 382 387— 428 387 4ll 417 425 425 429—478 429 458 465 473 473 479 — 519 479 500 507 515 S15 520 — 553 302 312 317 325 326 554 — 599 $54 573 580 590 591 600 — 636 600 615 620 627 628 a. 22, By 24, 15. 26. 27, (viii) Single Phase Motors Chapter Overview Short Answer Questions Objective Questions Answers to Objective Questions Hints to Selected Objective Questions Alternators Chapter Overview Short Answer Questions Objective Questions Answers to Objective Questions Hints to Selected Objective Questions Synchronous Motors Chapter Overview Short Answer Questions Objective Questions Answers to Objective Questions Hints to Selected Objective Questions Part-II : Power System Generation of Electrical Energy Chapter Overview Short Answer Questions Objective Questions Answers to Objective Questions Hints to Selected Objective Questions Economics of Power Generation Chapter Overview Short Answer Questions Objective Questions Answers to Objective Questions Hints to Selected Objective Questions Supply Systems Chapter Overview Short Answer Questions Objective Questions Answers to Objective Questions Hints to Selected Objective Questions Overhead Lines Chapter Overview Short Answer Questions Objective Questions Answers to Objective Questions Hints to Selected Objective Questions 637 — 657 637 646 649 654 678 — 700 701 — 724 701 712 74 Tis " 720 725— 752 725 737 P 7Al we 746 747 153 — 166 753 759 761 764 5 764 767 — 803 767 787 790 798 799 28. 29, 30. 31. Distribution of Electric Power Chapter Overview Short Answer Questions Objective Questions Answers to Objective Questions Hints to Selected Objective Questions Faults in Power System Chapter Overview ‘Short Answer Questions Objective Questions Answers to Objective Questions Hints to Selected Objective Questions Switchgear Chapter Overview Short Answer Questions Objective Questions Answers to Objective Questions Hints to Selected Objective Questions Protection of Power System Chapter Overview Short Answer Questions Objective Questions Aniswers to Objective Questions Hints to Selected Objective Questions (a) 855 — 882 855 869 875 880 a 880 883 — 899 883 891 894 898 898 1 Basic Concepts CHAPTER OVERVIEW 1. NATURE OF ELECTRICITY According to Modem electron theory of matter, all matter whether solid, liquid or gas is composed of very small particles called molecules. A molecule is in tum made up of atoms. An atom consists of ‘a central part called nucleus and around the nucleus (called extra-nucleus), there are a number of electrons revolving in different paths or orbits. The size of the nucleus is very small as compared to the size of the atom. The nucleus contains protons and neutrons. A proton is a positively charged particle having mass 1837 times that of an electron. A neutron has the same mass as proton but no charge. Clearly, the nucleus of an atom bears a positive charge. An electron is a negatively charged particle having negative charge equal to the positive charge on a proton. Under normal conditions, the number of electrons is equal to the number of protons in an atom. Therefore, an atom is neutral as ‘a whole, the negative charge on electrons cancelling the positive charge on protons, ‘The above discussion shows that matter is electrical in nature i.e. it contains particles of electricity viz protons and electrons. Whether a given body exhibits electricity (ie. charge) or not depends upon the relative number of these particles of electricity. (i) If the number of protons is equal to the number of electrons in a body, the resultant charge is zero and the body will be electrically neutral. Thus the paper of this book is electrically neutral (i.e. paper exhibits no charge) because it has the same number of protons and electrons. (ii) If from a neutral body, some *electrons are removed, there occurs a deficit of electrons in the body. Consequently, the body attains a positive charge. Hence a positively charged body has deficit of electrons from the normal due share. (iii) Ifa neutral body is supplied with electrons, there occurs an excess of electrons. Consequently, the body attains a negative charge. Hence a negatively charged body has an excess of electrons from the normal due share. 2. UNIT OF CHARGE The charge on an electron is so small that it is not convenient to select it as the unit of charge. In Practice, coulomb is used as the unit of charge. One coulomb of charge is equal to the charge on 625 x 10'®electrons i.e. 1 Coulomb = Charge on 625 x 10" electrons ‘Thus when we say that a body has a positive charge of 1 coulomb (1 C), it means that it has a deficit of 625 x 10" electrons from the normal due share. 3. FREE ELECTRONS ‘We know that electrons move round the nucleus of an atom in different orbits. The electrons in the last orbit are called valence electrons. In certain substances, especially metals (e.g. copper, aluminium * Electrons have very small mass and, therefore, are much more mobile than protons. On the other hand, protons are power-fully held in the nucleus and cannot be removed or detached, _ 2 Objective Electrical Technology etc) , the valence electrons are so *weakly attached to their nuclei that they can be easily removed or detached. Such electrons are called free electrons. It may be noted here that all valence electrons in a metal are not free electrons. It has been found that one atom of metal can provide at the most one free electron. Since a small piece of metal has billions of atoms, one can expect a very large number of free electrons in metals. Forexample, 1 cm’ of copper has about 8.5 x 10” free electrons at room temperature, 4, ELECTRIC CURRENT The flow of free electrons (or charge) in a definite directions is called electric current, The flow of electric current is shown in Fig. 1.1. The copper strip has a large number of free electrons. For simplicity, only the valence orbits are shown because only the valence electrons can take part in the flow of current, When electric pressure or voltage is applied, the free electrons being negatively charged start moving towards the positive terminal round the circuit as shown in Fig. 1.1. This directed flow of electrons is called electric current. Fig. Ll Conventionally, the direction of clectric current is taken along the direction of motion of positive charges. When current is caused by electrons (e.g in metals), the direction of current is opposite to the direction of electron flow. Note. It is important to note that none of the practical consequences nor any of the results of computations performed in the-study of electricity and electronics are in any way affected by the direction of current flow that one assumes. In this book, the direction of conventional current will be assumed, 5. MEASUREMENT OF CURRENT ‘The flow of charge in a definite direction is called electric current. It is measured by the time rate of flow of charge through the conductor. If q is the charge flowing through any cross-section ! of the conductor in time 1, then, E Electric current, red Fig. 1.2 * Ona relative scale, the spacing between the nucleus and valence electrons is vast. If a copper atom could ‘be magnified until the electrons were as large as coins, the valence electrons would be several kilometres away from the nucleus. Thisrelatively large distance dictates that valence electron is only weakly attached to the nucleus. Basic Concepts 3 If the rate of flow of charge varies with time, then current at any time (instantaneous current) is givenby; i. dt where dqis the small charge passing through any cross-section of the conductor in small time dt. The SI unit of electric current is ampere. If q= 1 C and f= 1 s, then [= 1/1 = | ampere. One ampere of current is said to flow through a wire if at any section one coulomb of charge flows in one second, Ifn electrons are passing through any cross-section of the wire in time f, then, le aan where e=—1.6X 10°C 6. ELECTRIC POTENTIAL Just as a body raised above the ground has gravitational potential energy, similarly, a charged body has electric potential energy. When a body is charged, work is done in charging the body. This work done is stored in the body in the form of electric potential energy. The charged body has the capacity todo work by moving other charges either by attraction or repulsion. Quantitatively, electric potential is defined as under } The electric potential at a point is the electric potential energy per unit charge. Electric potential energy _ W Vs Charge =o The SI unit of energy or work is 1 J and that of charge is 1 C so that SI unit of electric potential is 1 J/ C which is also called / volt. ‘Thus when we say that electric potential at a pointis 10 V, it means that if we place a charge of 1 C at that point , the charge will have electric potential energy of 10 J. Similarly, if we place a charge of 2 C at that point, the charge will have electric potential energy of 20 J. Note that potential energy per unit charge (i.¢ electric potential) is 10 V. 7. POTENTIAL DIFFERENCE ‘The difference in the potentials of two charged bodies is called potential difference (p.d.). Consider two bodies A and B having potentials of +5 V and +3 V respectively as shown in Fig. 1.3 (i). Each coulomb of charge on body A has an energy of 5 Joules while each coulomb of charge on body Bhas an energy of 3 Joules. Clearly, the body A is at higher potential than body B. A 8 A B (=) Ce) Gos Cn) w (ii) Electric potential, Fig. 1.3 Ifthe two bodies are joined through a conductor [See Fig. 1.3 (ii)], then *electrons will flow from body B tobody A, When the two bodies attain the same potential, the flow of current stops. Therefore, we arrive at a very important conclusion that current will flow in a circuit if potential difference exists. No potential difference, no current flow. It may be noted that potential difference is sometimes called voltage Unit. Since the unit of electric potential is volt, one can expect that the unit of potential difference will also be volt. Itis defined as : * The conventional current flow will be in the opposite direction ie. from body A to body B. 4 Objective Electrical Technology The potential difference between two points is 1 Volt if one joule of work is done in transferring 1 C of charge from the point of lower potential to the point of higher potential. Consider points A and B in an electrical circuit as shown in Fig. 1.4. Suppose V,— V, = 1 volt. It means that 1 J of work will be done in transferring 1 C of A B charge from point B to point A, Alternatively, 1 J of work (or energy) will be released (as heat) if 1 C of charge moves from ———1V—_>] point A to point B. Note that volt is the unit of energy. Fig. 14 8 MAINTAINING POTENTIAL DIFFERENCE ‘A device that maintains potential difference between two points is said to develop electromotive force (e,m,f.). A simple example is that of acell. Fig. 1.5 shows the familiar voltaic cell. It consists ofa copper plate (called anode) and a zinc rod (called a cathode) immersed in dilute H,SO,. The chemical action taking place in the cell removes electrons from copper plate and transfers them to the zinc rod. This transference of electrons takes place through the agency of dil. H,SO, (called electrolyte). Consequently, the copper plate attains positive charge of + coulombs and zinc rod a charge of —Q coulombs. The chemical action of the cell has done certain amount of work (say Wjoules) to do so. Clearly, the potential difference between the two plates will be WIQ volts. If the two plates are joined through a wire, some electrons from zine rod will be attracted through the wire to copper plate. The chemical action DIlLH,SO, of the cell now transfers an equal amount of electrons Fig. 15 from copper plate to zinc rod internally through the cell to maintain original potential difference (i.e. W/Q). This process continues so long.as the circuit is complete or so long as there is chemical energy. The flow of electrons through the external wire from zinc rod to copper plate is the electric current. ‘Thus potential difference causes current to flow while an e.m,f. maintains the potential difference, Although both e.m,f. and p.d. are measured in volts, they do not mean exactly the same thing. 9. CONCEPT OF E.M.F. AND POTENTIAL DIFFERENCE There is a distinct difference between e.m.f. and potential difference. The e.m.f. of a device, say a battery, is a measure of the energy the battery gives to each coulomb of charge. Thus if a battery supplies 4 joules of energy per coulomb, we say that it has an e.m.f. of 4 volts. The energy given to each coulomb in a battery is due to the chemical action. — al The potential difference between two points, say A and B, is a measure of the energy used by one coulomb in moving from A to B. Thus if potential difference between points A and B is 2 volts, it means that each coulomb will gave up an energy of 2 joules in moving from A to B. 10. OHM’S LAW ‘The relationship between voltage across and current through a conductor was first discovered by German scientist George Simon Ohm. This relationship is called Ohm's law and may be stated as : The current (1) flowing through a conductor is directly proportional to the potential difference (V) across its ends provided the physical conditions (temperature, strain, etc.) do not change ivt., Basic Concepts 5 lev or + = constant = R where R is a constant of proportionality and is called resistance of the conductor, For example, if in the Fig. 1.6 (i), the p.d. between points A and B of the conductor is V and current flowing is Z, then V/[ will be constant and equal to R, resistance of the conductor between points A and B. If Vis doubled up, current will also be doubled up so that ratio Vis constant. ie << v4 @ Fig. 1.6 Ifa *graph is drawn between applied potential difference (V) and current (J) flowing through the conductor, it will be a straight line passing through the origin as shown in Fig, 1.6 (#). Note that slope of the graph gives the resistance of the conductor (tan @ = V/ = R). 11. RESISTANCE The resistance of a conductor is defined as the ratio of p.d, applied across its ends to the resulting current through v the conductor ie oe & i ae Resistance is an opposition to the flow of current. If the resistance of the circuit is doubled, the current is reduced to one half. If the resistance is tripled, the current i is reduced to one-third and so on. The SI unit of p.d. is 1 Fig. 1.7 ‘Vand that of currentis 1 A. Therefore SI unitof resistance is V/A which has been given a special name ohm (symbol 2). ohm = 12=1V/A Aconductor is saidto havea resistance of 1 ohm if.a p.d. of | V across its ends causes a current of 1 A to flow through it. Use of resistance. Resistance can be very useful when the flow of current is to be controlled. — For example consider the circuit shown in Fig, 1.8. A component called rheostat has been added to the _i motor circuit. A rheostat is an adjustable resistor, The current flow must overcome the resistance of the circuit. As the rheostat is adjusted for more resistance, the circuit current J decreases and the motor slows down. As the rheostat is adjusted for Fig. 1.8 Rheostat ‘Taking Valong Y-axis and J along X-axis 6 Objective Electrical Technolgy less resistance, the circuit current increases and the motor speeds up. Thus we see that resistance can beused to control the speed of motor. Resistance can also be used to dim lights, control loudness and perform many other useful circuit functions. 12. CALCULATING RESISTANCE ‘The resistance R of a material of length / and area of cross-section A is given by : A where 9 (Greek letter ‘Rho’) is called resistivity or specific resistance of the material. Its value depends upon the nature of the material and temperature. 13. RESISTIVITY OR SPECIFIC RESISTANCE R=0 4 If!=1m;A=1m’, then R=p Hence specific resistance (or resistivity) of a material is the resistance offered by 1 m length of wire material having area of cross-section of I mi (See Fig. (# ————1 mn} 1.9). Fig 19 ‘The SI unit of resistivity is ohm-m (Q m). Different materials have different resistivities. For example, the resistivity of copper is 1:7 10°92 m. It means that if you take a copper wire I m long and having an area of X—section of 1 m’, then resistance of this piece of copper wire will be 1-7 x 10°Q. 14. CONDUCTANCE The reciprocal of resistance of a conductor is called its conductance (G). If a conductor has a resistance R, then its conductance G is given by : A circuit with high conductance has low resistance, and acircuit with low conductance has high resistance. The SI unit of conductance is siemen. It is denoted by the symbol S. Suppose a wire has a resistance of 0.5 Q. Then its conductance is given by : Conductivity. The reciprocal of resistivity of a conductoris called its conductivity. Itis denoted by the symbol o. If'a conductor has resistivity p, then its conductivity is given by : a Geo- Clearly, SI unit of conductivity is siemen per metre (Sm). 15. CARBON RESISTORS A component whose function in a circuit is to provide a specified value of resistance is called a resistor. The most commonly used resistors in electrical and electronic circuits are the carbon resistors. A carbon resistor is made from powdered carbon mixed with a binding material and baked into a small tube with a wire attached to each end. These small-sized resistors are manufactured in values from a fraction of an ohm to several million ohms. Note that the power rating of a carbon resistor Basic Concepts 7 depends upon the physical size of the resistor. A larger resistor is able to throw off (dissipate) more heat than a smaller one. Colour code for carbon resistors: Since a carbon resistor is physically quite small, itis more convenient to Fig, 1.10 use a colour code indicating the resistance value than to imprint the numerical value on the case. In this scheme, there are generally four colour bands A, B, C and D printed on the body of the resistor as shown in Fig. 1.10. The first three colour bands (A, B and C) give the value of the resistance while the fourth band (D) tells about the *tolerance in percentage. The table below shows the colour code for resistance values and colour code for tolerance. Colour Code for Resistance Values ‘Colour Code for Tolerance Black 0 Gren 5 Gold + 5% Bown 1 Blue 6 Silver + 10% Red 2 Viokt 7 Nocolour + 20% Orange 3 Gry 8 ‘Yellow 4 White 9 (To read the resistance value, we refer to the first three colour bands (A, B and C). The first two colour bands (A, B) specify the first two digits of the resistance value and the third colour band (C) gives the number of zeros that follow the first two digits. Suppose the first three colour bands (A, B,C) on the resistor are red, brown, orange respectively. Then value of the resistance is 21,000. Red ‘ & Brown : 1 2. Value=21,0002 Orange : 000 (ii) The fourth band D gives the value of tolerance in percentage. If colour of the fourth band is gold, tolerance is + 5 percent and if silver, then tolerance is + 10 percent. If the fourth band is omitted, the tolerance is assumed to be + 20 percent. ; a 3 : 4 ' ~ ery a im Black sib Rede" orsnge lO” — Green Blue chi Grey White Note: In order to remember the colour code, the above sentence may be helpful. 16. EFFECT OF TEMPERATURE ON RESISTANCE It has been found that in the normal range of temperatures, the resistance of a metallic conductor increases linearly with the rise in temperature. Therefore, resistance/temperature graph is a straight line as shown in Fig. 1.11. Consider a metallic conductor having resistance Ryat 0°Cand R, at °C. Then in the normal range of temperatures, the increase in resistance (i.., R, Ry) (® is directly proportional to the initial resistance, i.e., Fig. 1.11 * Due to manufacturing variati Thus, a resistor marked 100 is, the resistance value may not be the same as indicated by colour code. 10% tolerance means that resistance value is between 90 Q and 110 Q. 8 Objective Electrical Technology (ii) is directly proportional to the rise in temperature, i.e. Ri~ Ry = (iii) depends upon the nature of the material. Combining the first two, we get, Ri Ry = Rot or Ri-Ry = ORyt, wal) where a. is a constant of proportionality and is called temperature co-efficient of resistance. Its value depends upon the nature of the material and temperature. Rearranging eq. (i), we get, Ry= Ry (1+ 0t,) Definition of a: From eq. (i), we get, a = ie i increase in resistance/ohm original resistance/C rise in temperature Hence, temperature co-efficient of resistance of a conductor is the increase in resistance per ohm original resistance per °C rise in temperature. A litle reflection shows that unit of a will be ohms/ohms x °C i.e. /°C. Thus copper has a temperature co-efficient of resistance of 0-004/C. It means that if a copper wire has a resistance of 1 Qat 0°C, then it will increase by 0-004 Q for 1°C rise in temperature, i.e., it will become 1-004 Q at 1°C. It may be noted that metals have positive temperature co-efficient of resistance while semiconductors and insulators have negative temperature co-efficient of resistance. Note. If the resistance of a conductor is R, at f, °C and R, at 1, °C (t, <¢,), then, R= Rl+ay-4)1 17, OHMIC AND NON-OHMIC CONDUCTORS There are two types of conductors viz (i) ohmic conductors and (ii) non-ohmic conductors. (® Ohmic conductors. Those conductors which obey Ohm's law (fe V) are called ohmic conductors e.g. metals. The V — I graph for such a conductor is a straight line passing through the origin. Fig. 1.12(i) shows V-I graph for two ohmic conductors namely 1 kQand 2kQ resistors. The 1-kQ resistor conducts 5 mA at 5 V, 10 mA at 10 V. The 2 k@ resistor conducts 2:5 mA at 5 V and § mA at 10 V. 1 1 Ratko V= 10 volts Miliamperes Fig. 1.12 (i) Non-ohmic conductors. Those conductors which do not obey ohm's law (/ « V) are called non-ohmic conductors ¢.g, vacuum tubes, transistors, electrolytes etc. The V-I graph for Basic Concepts 9 such a conductor is non-linear. Fig. 1.12 (ii) shows ohm-ampere graph of a non-chmic conductor for a fixed p.d. of 10 V. A 1 kQ conductor conducts 10 mA, a 2 kQ conductor conducts 5 mA and a 4 kQ conductor conducts 2-5 m A. Note that relationship between resistance and current is non-linear. 18. ELECTRIC POWER The power of an electric appliance is the rate at which electrical energy is converted into other forms of energy (e.g. heat, etc.). For example, a 60 W bulb converts. gait a =e 60] of electrical energy into heat and light each second. ‘Thus referring to Fig. 1.13, as the charge q (= J 1) moves from point A to B, it loses electric potential energy = qV. In other words, qV joules of electrical energy is converted into Fig. 1.13 heat in ¢ seconds. —$—_ y ———— Electric Power = 2¥ aay = VI Sis o watts Electric Power = VJ watts = = PRwatts (sV=IR) = (ii) = VIR watts iii) Any one of the three formulas can be used to calculate electric power, depending upon the problem in hand, . Unit of Electric Power P=VI The SI unit of p.d. is 1 V and that of current is 1 A so that SI unit of power = 1 Vx 1 A=1 VA or 1 watt (1W). Hence electric power of a circuit or device is one watt if a current of I A flows through it when apd. of 1 V is maintained across it. The bigger units of electric power are kilowatt (kW) and megawatt (MW). 1kW = 1000W; = IMW=10°kW=10°w. Note. Electric appliances are rated in terms of electrie power. The faster the appliance converts electrical ‘energy into some other from of energy, the greater the electric power it has. Thus, in 1 second, a 100 W bulb converts more electrical energy into heat and light than a 60 W bulb. 19. ELECTRICAL ENERGY The loss of electrical potential energy in maintaining current in a circuit is called electrical energy consumed in the circuit. Thus in Fig. 1.13 above, as the charge q (= / 1) moves from point A to B, it loses electric potential energy = V= V1 joules. This loss of electric potential energy is converted into heat. We say that electrical energy consumed in t seconds is Vit joules. 2 Electrical energy consumed, W=VIt=PRr= + t joules Unit of Electrical Energy W = Vit=power x time The SI unit of power is | W and that of time is 1 s so that SI unit of electrical energy = 1 W x 1 s=1Wsoril. IJ (or 1Ws) energy is consumed when a device (e.g., bulb, heater, etc.) converts electrical 10 Objective Electrical Technology energy to other forms at a rate of 1 W for a time of I second. Commercial Unit: In practice, electrical energy is measured in kilowatt-hour (kWh). 1 kWh energy is consumed when a device converts electrical energy to other forms at a rate of TW fora time of 1 hour. Electrical energy inkWh = Power in kW x Time in hours ‘The electricity bills are made on the basis of total electrical energy consumed by the consumer. The unit for billing of electrical energy is 1 kWh. Thus when we say that a consumer has consumed 100 units, it means that electrical energy consumption is 100 kWh. 20. USE OF POWER AND ENERGY FORMULAS Ithas already been discussed that electric power as well as electrical energy consumed can be expressed by three formulas. While using these formulas, the following points may be kept in mind : 2 (Electric Power, P=PR= + watts Electrical energy consumed, W = FP Rr= ¥ i joules The above formulas apply only to resistors and to devices (e.g., electric bulb, heater, electric kettle etc.) where all electrical energy consumed is converted into heat. (ii) Electric power, P = Vi watts Electrical energy consumed, W = VJ joules These formulas apply to any type of load including the one mentioned in para (i). 21, ELECTRICAL MATERIALS ‘The materials used in electricity and electronics can be broadly divided into three major types viz 1. Conductors 2. Semiconductors 3. Insulators Conductors (e.¢ copper, aluminium etc.) conduct current very easily while insulators (e.g. glass, mica, paper) practically conduct no current. In other words, conductors have small resistivity and insulators have high value of resistivity. The resistivity of semiconductors (e.g. germanium, silicon etc.) lies between conductors and insulators. 1, Conductors (® Conductors are formed by metallic bonds. These bonds are based on a structure of positive metal ions surrounded by a cloud of electrons. (ii) Conductors have positive temperature coefficient of resistance i.¢ their resistance increases with the rise in temperature and vice-versa (see Fig. 1.14]. Positive coefficient (conductors) Zero coefficient Resistance Negative coetficient (Insulators and semiconductors) ‘Temperature Fig. 114 (ii) Conductors are used to carry current in electric circuits. Basic Concepts 1” 2. Semiconductors (i) Semiconductors are formed by covalent bonds. (i) Semiconductors have negative temperature coefficient of resistance ie. their resistance decreases with the rise in temperature and vice-versa [See Fig. 1.14] (iii) Semiconductors are used in the manufacturing of electronic devices (e.g crystal diodes, transistors etc .) 3. Insulators () Insulators are predominantly covalent compounds. The valence electrons are tightly locked with neighbouring atoms and are not available to support the flow of current. (ii) Insulators have negative temperature coefficient of resistance i.e. their resistance decreases with the rise in temperature and vice-versa. (iii) Insulators are used to confine the current to the desired path. SHORT ANSWER QUESTIONS Q.1. Why does a positive charge attract a negative charge ? Ans. Itisthe tendency of every body to have minimum electric potential ie., to obtain zero potential. A positive charge means deficit of electrons while the negative charge indicates excess of electrons, Consequently, positive and negative charges attract each other to have minimum electric potential. Q.2. A wire is carrying current, Is it charged ? Ans. No. The current in a wire is due to the drifting of free electrons in adefinite direction. But the number of electrons in the wire at any instant is equal to the number of protons. Hence, the net charge on the wire is zero. Q.3. The thermal speeds of free electrons are very large. Inspite of these high speeds, why they fail 10 escape from the surface of a conductor ? Ans. The free electrons in a conductor are free only to the extent that they may transfer from one atom to another within the conductor. It is because the free electrons that start at the surface of aconductor find behind them the positive ions pulling them back and none pulling forward. Thus at the suface of a conductor, a free electron encounters forces that prevent it to leave the conductor surface. External energy is required for the free electron to escape the conductor surface, Q.4. What are the fundamental differences between em. and p.d. ? Ans. The following are the differences between e.m.f. and pd. : (@ The name e.m.f. at first sight implies that it is a force that causes current to flow, But this is not correct because it is not a force but energy supplied to charge by some active device such as a battery. (ii) EMF. maintains p.d. while p.d. causes current to flow. (iii) When we say that e.m-f. of a device (e.g.. a cell) is 2 V, it means that the device supplies an energy of 2 joules to each coulomb of charge. When we say that a p.d. between points A and B of a circuit (suppose point A is at higher potential) is 2 V, it means that each coulomb of charge will give up an energy of 2 joules in moving from A to B. QS. Is volt a unit of energy ? ‘Ans. Yes, volt is the unit of energy. It is because volt is the unit of electric potential, Blectric potential, v = Work done _ W Q @ 12 Q6. Qz. Ans. QB. 0.10. Ans. Qu. Q.12. Ans, a1. Ans. Q.14. Objective Electrical Technology Therefore, the unit of electric potential is Joules/C. Now, Joules/C is named as volt. Two wires of equal length, one of copper and the other of manganin have the same resistance. Which wire is thicker ? tap + Bon = Pam. Pea Pm ay A, Since the resistivity of copper (p,) is less than the resistivity of manganin (p,,), the area of X- section of manganin wire (A,,) will be more than the area of X-section of copper wire (A,). Hence, manganin wire will be thicker. Why are alloys constantan and manganin used to make standard resistances ? Itis because these alloys have high resistivity and negligibly small temperature co-efficient of resistance, The drift velocity of free electrons is very small. Why does room light turn on at once as the switch is closed ? The electric field is transmitted with the speed of light. As soon as the switch is closed, the electric field is at once established in the whole circuit. As a result, free electrons everywhere in the wire begin drifiting‘at once. . We know that a large number of free elecirons are present in metals. Why is no current established in the absence of electric field ? Inthe absence of electric field, the free electrons in the metal have random motions, i.e., free electrons move in all directions haphazardly. During motion, they collide with positive ions of the metal again and again and after each collision, their direction changes. The result is that the net motion in any particular direction is zero. However, when an electric field is applied, the free electrons experience a force and start drifting towards the positive terminal of the source with a small velocity (drift velocity). Is ohm’s law true for all conductors ? No, it is true only for metallic conductors provided physical conditions do not change. Does the formula V = IR define ohm's law ? No. This formula defines resistance and can be applied to any conducting device whether or not it obey’s ohm's law. The essence of ohm's law is that the graph between V and I is linear i.e, the value of R is independent of V. Can we verify Ohm's law by using a filament lamp ? No. As the voltage across the lamp is varied, the temperature of the filament also changes. This in turn changes the resistance of the filament. ‘The essence of Ohm's law is that relation ‘between V and /is linear if R is independent of the magnitude of V. Why do conductors have positive temperature co-efficient of resistance ? The rise of temperature of a substance causes two effects : (@ It causes the molecules to vibrate more rapidly, thus impeding the movement of free electrons through the substance, (ii) It may break some bonds, releasing free electrons. With the rise in temperature of a conductor (e.g., metals), there is no increase in the number of free electrons and the sole effect of temperature rise is to increase resistance due to the increased molecular vibration. Hence conductors have positive temperature co-efficient of resistance i.e., their resistance increases with the rise in temperature. Why do insulators have negative temperature co-efficient of resistance ? A temperature rise in insulators creates many more free electrons than existed in the cooler Basic Concepts 13 Q.15, Ans. Q.16. Ans. Q.17. Q. 18. Ans. Q.19. Ans, Q.20. state. Often this increase in the number of free electrons more than offsets the interference to the drift movement caused by the increased molecular activity. Hence resistance of insulators (and for the same reason that of semiconductors) decreases with the increase in temperature ie., insulators have negative temperature coefficient of resistance. Why do some high resistance alloys (Eureka , manganin etc) have almost zero temperature co-efficient of resistance ? In case of some high resistance alloys (e.g., Eureka, Manganin, Constantan etc.), the rise of temperature causes negligible increase in resistance i., they have almost zero temperature co-efficient of resistance. It is because the two effects of temperature (mentioned in answer to Q. 13) almost cancel each other. What is the importance of temperature/resistance graph of a conductor ? ‘The temperature/resistance graph of a conductor is a straight line as shown in Fig. 1.15. We can very easily find the value of temperature co-efficient of resistance from this graph. i F a = Ao But R,-R, = and 1 = rise in temperature = AB But BC/AB is the slope of temperature/resistance graph. Fig, 1.15 = Slope of temperature / resistance graph = % Likewise, a, = Slopeof serene resistance graph Note that slope of temperature/resistance graph is constant (the graph being a straight line). Why is G9 of a conductor more than its Oyy? ‘The slope of temperature/resistance graph is constant and Ry (i.e., resistance at 0°C) is less than Rjo (ée., resistance at 10°C). Hence df, of a conductor is more than 01,4. How is the rise in temperature of the winding of a machine determined ? The resistance of the winding is measured both before and after the test run. Let R, and 1, be the resistance and temperature before the commencement of the test. After the operation of the machine for a given period, let these values be R, and f,. Now = R, = Ry (1 +Oo4) and R,=Ry (1+ yt) R _ lteter, R | 149% {t) Since R, and R, can be measured and 1, (ambient temperature) and Oy are known, the value of 1, can be calculated from eq(i). ‘The average rise in temperature of the winding will be (=) °C. What happens if power rating of a machine is exceeded ? Every electrical equipment has power rating marked on its body. While the equipment is in operation, care should be taken that power rating is not exceeded otherwise the equipment may be damaged/burnt due to excessive heat. Way does a 100 watt bulb give more light than a 60 watt bulb ? ‘A 100 watt bulb means that it is giving out 100 joules of energy per second while a 60 watt bulb indicates release of 60 joules of energy per second. Clearly, a 100 watt bulb will give more light than a 60 watt bulb. 4 Qa. Ans, Q.22 Ans. Q.23. Ans. Objective Electrical Technology How is work done in an electric circuit ? When voltage is applied in a circuit, it causes current (i.e., movement of free electrons) to flow through it. Clearly, work is being done in moving the electrons in the circuit. What is the difference between an overload and a short circuit ? An overload means that the circuit is carrying more than the rated current. A short circuit is said to occur if the circuit carries an exceedingly high current. In general, if the circuit carries more than 10 times the rated current, then short circuit should be suspected. What does an electric circuit do ? The function of an electric circuit is to convert electrical energy into some other form of energy. For example, electrical energy is converted into heat energy by devising a suitable heater circuit. In fact, the innumerable uses of electricity have been possible only duc to the proper use and applications of electric circuits. Q.24, Is the formula V = IR true for non-ohmic conductors ? ‘Ans, Yes, This formula defines resistance and not chm’s law ? Q.25. Is matter electrical in nature ? Ans. Yes, matter is electrical in nature. It is because every matter is made up of atoms and every aiom contains charged particles viz protons and electrons. OBJECTIVE QUESTIONS 1, The diameter of an atom is about @ 10m Gi) 104m (ii) 107m () 10m 2. The number of compounds available in nature is ( 105 Gi 300 (ii) 1000 (iv) unlimited 3. The mass of a proton is ..... the mass of electron. (@) equal to Gi) less than (iii) about 1837 times (iv) 200 times. 4, The electrons in the last orbit of an atom are called (i) free electrons (ii) valence electrons (iv) thermionic electrons 5. If the number of valence electrons of an atom is less than 4, the substance is usually () aconductor (ii) an insulator (iii) a semiconductor (iv) none of the above 6. If the number of valence electrons of an atom is more than 4, the substance is usually (® asemiconductor (ii) a conductor (ii) bound electrons (ii) an insulator (iv) none of the above. 7. Ifthe number of valence electrons of an atom is 4, then the substance is usually (® a semiconductor (ii) an insulator (iii) a conductor (iv) none of the above 8 Onecoulomb of charge is equal to the charge on (@ 625 x 10' electrons (ii) 6.25 x 10'* electrons Gif) 62.5x 10" electrons (iv) 0.625 x 10"° electrons 9. 1 cm? of copper at room temperature has about (®) 200 free electrons (ii) 20x 10" free electrons (iii) 8.5 x 10” free electrons (iv) 3x 10* free electrons 10. The electric current is due to the flow of (® positive charges only (ii) negative charges only (iii) both positive and negative charges (iv) neutral particles only MW. EMA inacircuit @ increases circuit resistance Basic Concepts (ii) decreases circuit resistance (iii) maintains potential difference (iv) none of the above 12, The electric current in a metallic conductor is due to the flow of (i) electrons only (ii) positive ions only (iii) electrons and positive ions only (iv) neutral atoms only 13, Potential difference has the unit of (i) charge Gi) power (iii) energy (iv) none of the above 14. The electric current ina liquid is due to the flow of (i) electrons only (di) positive ions only (iii) negative and positive ions both (iv) electrons and positive ions both 1S. Electric currentis a (i scalar quantity (i) vector quantity (iii) sometimes scalar and sometimes vector (iv) number only 16. A billion electrons pass through a cross- section of a conductor in 10™’s. The current is (107A Gi) 16x 107A Gi) 2x10 A (iv) 26x 107A 17. The electric current in a discharge tube containing a gas is due to (@ electrons only (ii) positive ions only (iii) negative and positive ions both Gv) electrons and positive ions both 18. How long does it take 50 4C of charge to pass a point in acircuit if the current flow is 15 mA? (@ 3.33 x 107s (iii) 28 (iv) none of the above 19. The specific resistance of a wire depends upon @) its length (ii) its cross-sectional arca (i) 6x10" s 5 (iii) itsdimensions (iv) its material 20, The quantity of charge that will be transferred by a current flow of 10 A over L hour period is, () 10 (i) 36x 10°C (iii) 24x 10°C (iv) 16x 10°C 21. The resistance of a straight conductor is independent of (® temperature (ii) material (if) cross-sectional area (iv) shape of cross-section 22, A length of wire hasa resistance of 6 ohms. ‘The resistance of a wire of the same material three times as long and twice the cross- sectional area will be ( 36 ohms (ii) 9 ohms (if) 1 ohm (iv) 12 ohms 23, The drift velocity of electrons is of the order of @ Ims™ (ii) 107 mst (iit) 10° ms" (iv) 3x 10° ms 24. Conductivity is the reciprocal of (i) current density (ii) resistance (iii) resistivity (iv) none of the above 25. The specific resistance of a metallic conductor........ With rise in temperature (@ increases (ii) decreases (iii) remains unchanged (iv) none of the above 26. The potential difference of an energy source that provides 50 mJ of energy for every micro coulomb of charge that flows is @sv Gi) SOV (iii) 500 V. (iv) SOV 27. Insulators have ... .-». temperature co- efficient of resistance (i) zero (i) positive (iii) negative (iv) none of above 28. Eureka has ... temperature co-efficient of resistance, @ almost zero Gi) negative (iii) positive (iv) none of the above 16 29. What quantity of charge must be delivered by a battery with a potential difference of 100 V to do 500 J of work ? @s5c (i) OSC (iii) SOC (iv) 500C 30. A piece of aluminium wire is stretched to reduce its diameter to half of its original value. Its resistance will become () two times (ii) four times (ii) eight times (iv) sixteen times 31. Semiconductors have ....... temperature coefficient of resistance (® zero (i) negative (ii) positive (iv) none of the above 32. The thermal speed of electrons is of the order of @ 1ms* (i) 10° ms" (ii) 10° ms" (iv) 3x 10% ms 33. Constantan wire is used for making standard resistance because it has (i) low specific resistance (ii) high specific resistance (iii) negligibly small temperature co-efficient of resistance (iv) high melting point 34, ‘The resistance of a material 2 m long and 2 m’in area of x-section is 16 x 10° Q. Its specific resistance will be @ 16x10 Qm (i) 64x 107° 2m. (ii) 32x 10° Qm (iv) 0-16 10* Qm 35. The SI unit of conductivity is (@ ohm-m (i) ohm ii) mho-m (iv) mhofm 36. The value of o (temperature co-efficient of resistance) depends upon (i length of the material (ii) X-sectional area of the material (ii) volume of the material (iv) nature of the material and temperature 37. A copper wire of resistance R, is stretched till its length is increased n times of its original length. Its resistance now will be (wR, G) RY (iii) aR, (iv) WR, Objective Electrical Technology 38. The resistivity of a material is 2x 10° Qm. What will be the resistance of a hollow pipe of the material of length 1 m and having inner and outer radii 10 cm and 20 cm respectively ? ) 2x107Q ii) 21x107Q (ii) 3x10°Q (iv) 282 1072 39, A piece of aluminium (Al) and germanium (Ge) are called from 7, K to 7, K. The resistance of (® each of them increases (ii) each of them decreases (ii) aluminium increases and that of germanium decreases () aluminium decreases and that of germanium increases 40, At 0K, germanium behaves as (é) aconductor (ii) an insulator (iii) a superconductor (iv) a ferromagnetic substance 41, The example of non-ohmic resistance is (© copper wire (ii) carbon resistance (iii) aluminium wire (iv) tungsten wire 42. A copper wire is stretched so that its length is increased by 0.1%. The change in its resistance is i) 0.3 % (iii) 0.2 % (@) none of the above 43, If an electric current is passed through a nerve, the man (@ begins to laugh (iii) is excited (iv) becomes insensitive to pain 44, Which one of the following is the best conductor of electricity ? (@ silver Gd) gold (iii) copper (iv) zine 45, The temperature co-efficient of resistance of a wire is 0-00125 per °C, At 300 K, its resistance is 1 Q. The resistance of the wire will be 2 Qat @ 154K (ii) 1400 K Gi) 04% (di) begins to weep Gi) 100K (iv) 127K Basic Concepts 46. Ampere - hour is the unit of @ quantity of electricity (i) strength of current (iii) power (iv) energy 47, The resistance of aconductoris 5Q at 50°C and 6 Q at 100°C. What is its resistance at orc? @ 12 (i) 22 Gi) 3Q (iv) 4Q 48, Fig. 1.16 shows the temperature / resistance graph of a conductor. The value of cr, is (@ 0.005°C (i) 0.04°C i) O.1PC (iv) 04°C 49. Referring to Fig. 1.16, the value of ety, will bes. Fig. 1.16 @ 0-0057°C (ii) 0-004PC (iii) 047°C (iv) 005°C 50. Referring to Fig. 1.17, the value of a is Resistance (Q)—» Temperature» Fig. 1.17 (i) 1/30 per °C (ii) 1/40 per °C (iii) 1/1200 per °C (iv) none of the above 17 51. Referring to Fig. 1.17, the value of Ryg will be .. ( 70Q (i) 40Q (iii) 35Q (iv) 30 52. Referring to Fig. 1.17, the value of ct, will be. (i) 1/30 per °C (ii) 1/70 per °C (iii) 1/50 per°C (iv) 1/1200 per °C 53. The value of a, of a conductor is 1/236 per °C. The value of og will be.. - @ U2BWper°C (ii) 1/272 per °C (iif) 17254 per °C (iv) none of the above 54, The value of 45 of a conductor is 1/255 per °C. The value of oo will be (1300 per °C (ii) 1/230 per °C (iii) 1250 per °C (iv) 1/265 per °C 55. The value of 09 of a conductor is 1/230 per °C. The value of a, will be (@ 1/180 per °C (ii) 1/280 per °C (iii) 1250 per °C (iv) none of the above 56. The number of free electrons passing through the filament of an electric lamp in one hour when the current through the filament is 0-32 A will be @ 3x 107 (ii) 210% (ii) 72x10" (iv) 72x 107" 57. In which of the following substances the resistance decreases with increase in temperature ? (@) carbon (#) constantan (iii) copper (iv) silver 58. A wire having very high value of conductance is said to be (i) very good conductor (ii) moderately good conductor (iii) insulator (iv) none of the above 59. Anelectric heater is marked 2000 W , 200 V. The resistance of the coil is 012 (i) 20. (iit) 120 Q (iv) 200 60. The current in a circuit having constant 18 resistance is tripled. ‘The power increases @ $ times (i) 3times (iid) 9 times Go) } times 61. What voltage drop will be there across a 1 KW electric heater whose resistance when hot is 40.2? @ 100V i) SOV (ii) 150 V (iv) 200V 62. A resistor R, dissipates the power P when connected to a certain generator. If resistance R, is put in series with Rj, the power dissipated by R, @ decreases (ii) increases (iii) remains the same (iv) any of the above depending upon the values of R, and Ry 63. In case of liquids, ohm’s law is (i) fully obeyed (ii) partially obeyed (ii) there is no relation between current and p.d. (iv) none of the above 64. Two electric bulbs rated for the same voltage have powers of 200 W and 100 W. If their resistances are respectively R, and R,, then, @ R,=2R, (@ R,=2R, (ii) R,=4R, () R=4R, 65. A copper wire has a resistance of 10. Itis stretched by one-tenth of its original length. Then its resistance will be () 102 (i) 12.12 (ii) 9Q (iv) 1Q 66, The current-voltage graphs for a given metallic wire at two different temperatures T, and T; are shown in Fig, 1.18. Then Ts Fig. 1.18 Objective Electrical Technology @ T,>T, (ii) T, Pe (ii) Pa < Pp (iv) Information insufficient 78. As the temperature of a metallic resistor is increased, the product of its resistivity and conductivity (i) increases (iii) remains constant (iv) may increase or decrease A resistor develops 400 J of thermal energy in 10 s when a current of 2 A is passed through it. The resistance of the resistor is (i) 4.7K 2 420% (i) 1kKQ 4 20% GD P4= Pp (W decreases ANSWERS TO OBJECTIVE QUESTIONS @ 102 Gi) 202 1 @ 2 3. Gi 6. (iii) L@ 8. @ ML. (ii) 12. © 13. (iii) 16. (ii) 17. (iv) 18. @ 21. (iv) 22. (ii) 23. (ii) 26. (iv) 27, (it), 28. 31. Gi) 32. (iid) 33. (iii) 36. (iv) 37. @ 38. (ii) 41. (ii) 42. (iid) 43. (ii) 46. @ 47. (iv) 48. () sl. 52. (ii) 53. (ii) 56. (iv) 57. (i) 58. () 61. (>) @ 8. @ 62. 19 (i) 5Q (iv) 40 80. An electron moves in a circle of radius 10cm with a constant speed of 4 x 10° ms". The electric current at a point on the circle is @ 1x10 (i) 4x 107A (iii) 2x 107° A (iv) none of above 81. The resistance of human body is about 12 @ 2002 Gi) 10Q (ii) 1000 2 (i) 252 82. The filament of an electric bulb is made of () tungsten (i) nickel (iii) steel (@) aluminium 83. The hot resistance of electric bulb’s filament is higher than its cold resistance because the temperature co-efficient of filament is @ zero () positive Gii) negative (i) none of above 84. A nichrome wire used as a heater coil has a resistance of 2 Q/m. For a heater of | kW at 200 V, the length of the wire required is @ 80m (i) 40m (ii) 20m (iv) 24m 85. When | Vis applied in a circuit a current of 1 fA flows through it. The conductance of the circuit is (@® 1 4mho (ii) 2 pmbo i) 10° mho (iv) none of above 4, (iif) 5. @ 9 (ii) 10. (iii) 14. (iii) 15. 19. (iv), 20, (ii) 24, (iii) 25. 29. @ 30. Civ) 34. 0 35. (iv) 39. (iv). 40. (ii) 44. (0 45. (ii) 49. id) 50. () 54, (iii) 55. 59. (ii) 60. (iii) 64. (if) 65. (ii) 66. (i) 71. (ii) 76. (ii) 81. Gi) 16. 1 18. t 20. q 22. R ay RX or Ry 26. v 29. Q 30. R, & R Now, 4a, & R 37. Ry Now 1 Se aye Rk R or R 38. R Objective Electrical Technology 67. (iii) 68. (ii) 69. (iv) 70. (i) 72. (wv) 73. (iv) 14. © 75. (i) TI. Ww) 78. (iii) Dp. @ 80. 82. () 83. (ii) 84. (iii) 85. () HINTS TO SELECTED OBJECTIVE QUESTIONS, 3 9 = Me IO X16xID 816x107 4 t 10 = 72 50x10% 4. 3 Fo pig 7333x107 s = 11=(10)x(60x60)=3-6x 10°C apt » eo =P 5 Oy w (e)(3)-C)+G5)- 1.5R,=1.5x6=9 ohms 3 = W. 0x10" © 50x 10° V=50KV Q 1x10 w <= i! mie ag a a " > cH aE x gs fe tap “ K = a 8 g = 1 aN oP oe ” s = (nia or azayin Basic Concepts 21 Herer p= 2x 10° Qm;1= Im;a=(r7 - #)=n[(0-2° ~ (0-17 ]= 9-42 x107 m? R= 2x10% x—__, 221x107 @ 9-42 x10 1. =ph, 42. R=p7; R07 Ol ' = 1+——xi=1-0011 Now / 100 As volume remains the same, al = a’l’ or a’ = 1 Re (©) (2) = (009 (01) = 1-02 R t or eee = 0002 s+ Percentage increase = Res X 100 = 0-002 x 100 = 0-2% 48. a, = Slope of temp/resistance graph _ 10/50 R 0. c, = ope of temp//resistance graph i ee zo 53, a, = —%_-.—1_ 1.1L 18 Taq x18 (1/eq) +18 236 +18 asa © = Lake 56. ie ee It _ 0-32. 3600 21 nos He OSE 27-210 e 16x10" 2 a 61. pak V = VPR = {100% 40 = 200 volts 3% = 6% Violet Orange Silver f 47 x 108 + 10% = 47kQ2 10% 2 Objective Electrical Technology 76. Brown Black Red None \/ 10 x 10 + 20% = 1kQ*20% 79. Energy = PRr Energy _ __400. =" =102 Pe yx 10 80. The number of revolutions made by the electron in one second 4x10° Charge crossing the point on the circle per second Ixio"? A = (x 10") xa-6x 107% mt 2 D.C. Circuits CHAPTER OVERVIEW 1, DIRECT CURRENT ‘The current that always flows in one direction is called direct current (d.c.). The current supplied by acell/battery or d.c. generator is direct current. Thus in Fig. 2.1, the battery supplies direct currentto the bulb. The direction of current is along ABCDA and it always flows in this direction. Note that direct current means steady direct current (i.e. one of constant magnitude) unless stated otherwise. 2. D.C. CIRCUIT The closed path followed by direct currentis called a d.c. circuit. A de. circuit essentially consists of a source of direct voltage (c.g. battery), the conductors used to carry current and the load, Fig. 2.1 shows a torch bulb (i.e. load) connected to a battery through conducting wires. The direct current™ starts from the ‘| aa, positive terminal of the battery and comes back to the starting Fig. 2.1 point via the load, The direct current follows the closed path ABCDA and hence ABCDA is a d.c. circuit. The load for a d.c. circuit is usually a **resistance. In a dic. circuit, loads (i.e. resistances) may be connected in series or parallel or series-parallel. 3. RESISTORS IN SERIES A number of resistors are said to be connected in series if the same current flows through each resistor and there is only one path for the current flow throughout. Consider three resistors of resistances Rj, R, and R, connected in series across.a battery of E volts as shown in Fig. 2.2 (i). Then total resistance R,is given by, B BULB c ! y Ry = R,+R,+R, , R, R. Ry i é Rr s a ' ' 44 4H E E @ (i) Fig. 2.2 * This is the direction of conventional current. However, electron flow will be in opposite direction. ** Other passive elements viz inductance and capacitance are relevant only in ac. circuits, 23 24 Objective Electrical Technology Hence when a number of resistances are connected in series, the total or equivalent resistance is equal to the sum of the individual resistances. Thus we can replace the series connected resistors shown in Fig. 2.2 (i) by a single resistor R,.(= R, + R, + R3) as shown in Fig. 2.2 (ii). This will enable us to calculate the circuit current easily (/ = E/R,). (i) When resistors are connected in series, the total circuit resistance increases. i) Ry = Rr or oe 1. or al where P,.is the total power dissipated by the series circuit and P,, P, and P, are the powers dissipated by individual resistors. 4. RESISTORS IN PARALLEL A number of resistors are said to be connected in parallel if voltage across each resistor is the same and there are as many paths for current as the number of resistors, Consider three resistors of resistances R,, R, and R, connected in parallel across a battery of E volts as shown in Fig. 2.3 (/). Then total resistance Ris given by ; ay 1 1 RR | | I] 1 E @® ii) Fig, 2.3 Hence when a number of resistances are connected in parallel, the reciprocal of the total resistance is equal to the sum of reciprocals of individual resistances. Again, we can replace the parallel connected resistors shown in Fig. 2.3 (i) by asingle resistor R, shown in Fig. 2.3 (ii). (@ When resistors are connected in parallel, the total circuit resistance decreases. (ii) The total resistance of a parallel circuit is always less than the smallest of the resistances. For example, if three resistors of 1 Q, 3 Q and 4 Q are connected in parallel, the total resistance will be less than 1 2. (iii) If n resistors, each of resistance R, are connected in parallel, then total resistance R= Rin. i 1 tid Ww) soto RRR 2 2 yr y2 y vv iy FI or FI » P ee DC. Circuits 25 or P, = P\+P,+P, where P,.is the total power dissipated by the parallel circuit and P,, P, and P, are the powers dissipated by individual resistors. 5, TWO RESISTORS IN PARALLEL A frequent special case of parallel resistors is acircuit that contains Ps nwo resistors in parallel as shown in Fig, 2.4. The total circuit current I divides into two parts; I, flowing through R, and /, flowing through Ry. — 5. 1 1_R+Rk 1, Rp (i) Total Resistance: — +e = AWA Ry , RRR or R, = — 2 ie, R+R Sum * Thus, if two resistances of 3 Q and 6 Q are connected in ——|[}] parallel, then their total or equivalent resistance R is E Fig. 2.4 = 3X6_ 1839 © 3+6 9 (ii) Branch Currents: = E = E_{;_®& Now 1, = £e\1 OR ( R+R RtR, nage Similarly, I, = 1x RoE i.e.; current in any of the two branches = Total current x Other resistance _ Sum of the two resistances ‘Thus referring to Fig. 2.5, the currents in the hy sa two branches are : ga i, = 9x =6a mH 3+6 : 4 sa hs IRs gees Fig. 25 6. ADVANTAGES OF PARALLEL CIRCUITS The most useful property of a parallel circuit is the fact that potential difference has the same value between the terminals of each branch of parallel circuit. This feature of the parallel circuit offers the following advantages : (The appliances rated for the same voltage but different powers can be connected in parallel without disturbing each other's performance. Thus a 230 V, 230 W TV receiver can be operated independently in parallel with a 230 V, 40 W lamp. (i) If a break occurs in any one of the branch circuits, it will have no effect on other branch circuits, Due to above advantages, electrical appliances in homes are connected in parallel. We can switch on or off any light or appliance without affecting other lights or appliances 26 Objective Electrical Technology 7, INTERNAL RESISTANCE OF A CELL The resistance offered by a cell to the currént flow is called internal resistance r of the cell. The internal resistance of a fresh cell is generally low. However, as the cell is used, its internal resistance goes on increasing. The internal resistance of a cell depends upon the following factors : (distance between the plates — increases with the increase in distance between the plates (i) nature of the electrolyte, (iii) concentration of the electrolyte - increases with the increase in concentration of electrolyte. (iv) nature of the electrodes (v) area of the plates ~ decreases with the increase in plate area. It is a usual practice to show intemal resistance of a cell as a series E fr resistor external to the cell as shown in Fig. 2.6. rs 8 EM. AND TERMINAL VOLTAGE OF A CELL Fig 26 (#) When the cell is delivering no current (i.¢., on open-circuit), the p.d. across the terminals of the cell is equal toe.m.f. E of the cell as shown in Fig. 2.7 (i). f e= et 1 AW. @ (ip Fig. 2.7 (ii) Whena resistance R is connected across the cell [See Fig. 2.7 (i/)], current / starts flowing in the circuit. This current causes a voltage drop (= / r) across internal resistance of the cell so that terminal voltage Vis less than the e.m.f. E of the cell. == Retr or IR+Ir =E But / R= V= Terminal p.d. of the cell 3 Vtlr aE or E=V+lr E-v Internal resistance of the cell, =r = 9. WHEATSTONE BRIDGE This bridge was first proposed by Wheatstone (an English telegraph engineer) for measuring accurately the value of an unknown resistance. It consists of four resistors (two fixed known resistances P and Q. a known variable resistance Rand the unknown resistance X whose value is to be found) connected to form a diamond shaped circuit ABCDA as shown in Fig. 2.8 (i). Across one pair of opposite junctions (A and C), battery is connected and across the other opposite pair of junctions (B and D),a galvanometer is connected through the key K, The circuit is called a bridge because galvanometer 0.C. Circuits 27 bridges the opposite junctions B and D. Fig. 2.8 (ii) shows another* way of drawing the Wheatstone 1 | BATTERY bridge BATTERY @ (i) Fig. 28 Let /, and /, be the currents through P and & respectively when the bridge is balanced. Since there is no current through the galvanometer, the currents in Q and X are also 1, and J, respectively. As the galvanometer reads zero, points B and D are at the same potential, This means that voltage drops from A to B and A to D must be equal. Also voltage drops from B to C and D to C must be equal. * LP = LR and 10 = 1X Be wll Oo” x or PX =QR uO) Le. Product of opposite arms = Product of opposite arms Unknown resistance, x= Sxr Hence when the Wheatstone bridge is balanced, the product of resistances of the opposite arms of the bridge are equal. Note that exp (i) is true only under the balanced conditions of Wheatstone bridge 10. KIRCHHOFF’S LAWS Sometimes we encounter circuits where simplification by series and parallel combinations is impossible. Consequently. Ohm's law cannot be applied to solve such circuits. Kirchhoff gave two laws to solve such complex circuits, namely : (i) Kirehhoff’s Current Law (KCL) (ii) Kirchhoff"s Voltage Law (KVL) ( Kirchhoff’s Current Law. This law relates to the currents at the **junctions ofan electric circuit and may be stated as under : The algebraic sum of the currents meeting at a junction in an electrical circuit is zero. An algebraic sum is one in which the sign of the quantity is taken into account. For example, consider tour conductors carrying currents /,, f,4, and /, and meeting at point O as shown in Fig. 2.9. If we take the signs of currents flowing towards point O as positive, then currents flowing away from ote the four points A, B, C and D, each lying at the junction between two resistors. A galvanometer should bridge a pair of opposite points such as B and D and the battery to the other pair A and C. A junction is that point in an electrical circuit where three or more circuit elements meet. 28 Objective Electrical Technology point © will be assigned negative sign. Thus, applying Kirchhoff's current law to the junction O in Fig. 2.9, we have, (1) + () + (Ch) + (Ch) = 0 or hth = hth ie, Sum of incoming currents = Sum of outgoing currents Hence, Kirchhoff’s current law may also be stated as under: Fig. 2.9 The sum of currents flowing towards any junction in an electrical circuit is equal to the sum of currents flowing away from that junction. Kirchhoft’s current law is rightly called the junction rule. Kirchhoff’s current law is true because electric current is merely the flow of free electrons and they cannot accumulate at any point in the circuit. This is in accordance with the law of conservation of charge. Hence, Kirchhoff’s current law is based on the law of conservation of charge. (ii) Kirchhoff’s Voltage Law (KVL). This law relates to e.m.fs and voltage drops in a closed circuit or loop and may be stated as under : Inany closed electrical circuit or mesh, the algebraic sum of all the electromotive forces (e.m.fs) and voltage drops in resisiors is equal to zero, i.e., In any closed circuit or mesh, Algebraic sum of e.mfs + Algebraic sum of voltage drops = 0 The validity of Kirchhoff’s voltage law can be easily established by referring to the loop ABCDA shown in Fig. 2.10. If we startfrom any point (say point A) Ry Re in this closed circuit and go back to this point (i.., point A) B ce after going round the circuit, then there is no increase or decrease in potential. This means that algebraic sum of the e.m.fs of all the sources (hete only one e.m-f. source is 41 considered) met on the way plus the algebraic sum of the voltage drops in the resistances must be zero. Kirchhoff"s e voltage law is based on the law of *conservation of energy, A" || D i.e., net change in the energy of a charge after completing Fig.210 the closed path is zero. Note. Kirchhoff's voltage law is also called loop rule. Sign convention. While applying Kirchhoff’s voltage law to a closed circuit, algebraic sums are considered. Therefore, it is very important to assign proper signs to e.1n.fs and voltage drops in the closed circuit. The following sign convention may be followed : A rise in potential should be considered positive and fall in potential should be considered negative, (® Thus in Fig. 2.11 @, as we go , -—|—_8 A o———_-4W—+— B from A to B (i.e., from negative terminal of the cell to the positive terminal), there oO “a is a rise in potential, In Fig. 2.11 (i), as Fig. 2.11 we go from A to B, there is also a rise in potential. ce——_|——ed c+» (i) In Fig. 2.12 (i), as we go from C > — to D, there is a fall in potential. In Fig. Fig. 212 ii) * As acharge traverses a loop and returns to the starting point, the sum of rises of potential energy associated with e.m.fs in the loop must be equal to the sum of the drops of potential energy associated with resistors. D.C. Circuits 29 2.12 (ii), as we go from C to D, there is again a fall in potential. Ul. BATTERY AND ITS NEED The e.m.f, and current obtained from a single cell is generally small. For instance, an ordinary dry cell has ane.m.f, of 1.5 V and can deliver about 1/8 ampere continuously. Such a cell can, therefore, supply electrical energy to a circuit requiring 1.5 V and not more than 1/8 A. Many occasions arise when higher voltage or higher current or both are required. To meet these needs, a number of cells are suitably connected or grouped. The combination of cells thus obtained is called a battery. Depending upon voltage and current requirements, cells may be connected in three ways to forma battery viz. (i) series grouping (#) parallel grouping (iii) series-parallel grouping. 12. CELLS IN SERIES When voltage required is more than that of the e.m-f. of a single R cell, a number of cells are connected in series to meet the requirement. The cells are said to be connected in series if the negative terminal of one cell is connected to the positive terminal | 4 ’ of the next cell and so on, Consider n cells, each of ¢.m.f. £ and intemal resistance r, | F connected in series across an external resistance R as shown in BATTERY Fig. 2.13. Fig. 213 Total battery emf. = nE Internal resistance of the battery Total circuit resistance nr Renr nE R+nr Circuit current, 1 Special Cases (i) IfR >> nr, then mr can be neglected as compared to R. i= £ = nx current due to one cell (i) If R << nr, then R can be neglected as compared to nr. nE_E f= Fp 7p = surrent due to one cell Hence in order to get maximum current in a series grouping of cells, the external resistance (R) should be very high as compared to the internal resistance of the batiery (n r). 13. CELLS IN PARALLEL When current required is more than that delivered by a single cell, a number of cells are connected in parallel to meet the requirement. In parallel grouping, positive terminals of all the cells are joined together and in a like manner all the negative terminals are connected together. Consider m rows of cells in parallel, each row containing one cell. Let E and r be the e.m.f. and internal resistance | , respectively of each cell. Further, let this battery be connected across an extemal resistance R as shown in Fig. 2.14. EMF of the battery = E Since the cells are connected in parallel, their internal Fig, 2.14 30 Objective Electrical Technology resistances are alsa in parallel. If r, is the total resistance of the battery, then, As te teh. miterms ™m or rr " _ LeL+ T+. terms _ om r r 1, = rim Total circuit resistance = R+1,=R+ 7 : E mE Cineuit current, = SE = eee m or "5 Re? Special Cases () If R<r (ii) R=r (iv) any other value of R 14, Five cells each of e.m.f, £ and internal resistance rare connected in series. If due to oversight, one cell is connected wrongly, then equivalent e.m.f. and internal resistance of the combination are () 38 and 5r (ii) SE and Sr (iii) 3 and 3r (iv) SE and 4r 15. A wire has aresistance of 12 Q. Itis bent in the form of a circle. The effective resistance between the two points on any diameter of (iii) (E-V)r the circle is (62 (i) 32 Gi) 129 (iv) 24Q 16. The smailest resistance obtained by connecting 50 resistances of 1/4 ohm each is () S0AQ (ii) 4/50 Q Gi) 200Q (iv) 1/200 Q 17. The internal resistance of a cell of e.m.f. B 2-0V is 0-1 Q. Itis connected to a resistance of 39. The voltage across the cell is @05V (ii) 1-95 V Gi) 19V (iv) 2V 18. The total conductance of the circuit shown in Fig. 2.22 is 102 22 12 Fig, 2.22 () 138 @) 6S Gi 6S (iv) 258 19. If 10 Q resistance is removed in Fig. 2.22, then total conductance of the circuit will be G) 3S (ii) 6S ii) 28 @) LSS 20, The voltage across the parallel circuit shown. in Fig. 2.23 is 192 158 252 2a Fig. 2.23 @ SV Gi) 10V (iii) 30.V (iv) 125 V 21. The current in 10 &2 resistor in Fig. 2.23 is 3A {ii) 2SA (iii) VS A {iv) SSA 22. Three 2 ohm resistors are connected to form atiangle. The resistance between any two comers is (i) 6Q Gi) 22 (iii) (3i4) Q Gv) 4B) Q 23. A cell of negligible internal resistance and e.m.f. 2 volts is connected to series combination of 2, 3 and 5 ohms. The potential difference across the terminals of 3 Q resistor will be @ @B)V (i) 3 (i) 06V @) 6V 36 24, A 200 W and 100 W bulb both meant for operation at 220 V are connected in series. When connected to a 220 V supply, the power consumed by them is () 66W (ii) 33. W (iii) 100 W (iv) 300 W 25. An electric fan and a heater are marked as 100 W, 220 V and 1000 W, 220 V respectively. The resistance of heater is (i) zero (ii) greater than that of fan (iii) less than that of fan (iv) equal to that of fan 26. Three cells each of e.m.f. 1-5 V and internal resistance 1 Q are connected in parallel. The e.m.f. of the combination is @45V (@5V (iii) 3V (iv) 15 V 27. When the internal resistance of a cell is large compared to the external resistance, then high current is obtained by grouping the cells in @ series (iii) series-paraliel (iv) all three can be used 28. The e.mf. of acell depends upon (@ intemal resistance (ii) external resistance Gii) electrolyte and electrodes of the cell (iv) none of these factors 29. Three equal resistors are connected as shown in Fig. 2.24. Find the equivalent resistance between points A and B. (i) parallel + Sn [sha tol ys Fig, 2.24 () 3R (i) RB (iit) 3RI2 (iv) 2R/3 30. Given three equal resistances, How many combinations of these three resistances can be made (0 three iii) five (ii) four (iv) two. Objective Electrical Technotogy 31, Identical wires of nichrome and copper are connected in series in a circuit. This results in (® greater current in nichrome (ii) greater current in copper (iii) greater heat in nichrome (iv) greater heat in copper 32. The effective resistance between B and Cof letter A containing resistances as shown in Fig. 2.25 is Fig. 2.25 @ 602 (i) 402 (ii) 1609.2 (iv) 803 2 33. Two resistances are in parallel and give equivalent resistance of 6/5 Q. One of the resistances is broken and the effective resistance is 2 Q. The resistance of the broken resistor is, @ 352 (i) 29 (i) 32 (iv) 62 34, In the circuit shown in Fig. 2.26, the final voltage drop across the capacitor C is Vv % tg Fig 2.26 @ VL Gy YO ith ith (iy YD) iy Mt) F aga 35, Two identical cells connected in series send 10 A through 5 Q resistor, When they are connected in parallel, they send 8 A through the same resistor. The internal resistance of each cell is @) zero Gi) 25Q D.C, Circuits (ii) 102 (&) 19 36. Fig, 2.27 shows currents in the part of an electric circuit. Then current iis 2A 1A V3A 2A A Fig. 2.27 @ LTA (ii) 37 A (iii) L3A (iv) LA 37. Fig. 2.28 shows a part of a closed circuit. What is the potential difference between points A and B ? 62 B ( 6v (@ RV (ii) UV (iv) 18V 38. Fig. 2.29 shows a part of a closed electrical circuit. Then V,—V,is 24 92 ,2V 12 a i Fig. 2.29 ( -8V (i) 6V Gif) 10 ti) 3V 39. A current of 2 A flows in a circuit shown in Fig.2.30. The potential difference V,~ Vz is Fig. 2.30 @-v (i) 41 (ii) 4V @) 2V 40. For what value of unknown resistance X, the potential difference between points B and D a7 will be zero in the circuit shown in Fig. 2.31 Fig. 2.31 (9 4Q ii) 22 (iii) 3Q (iv) 62 41. For what value of unknown resistance X, the potential difference between B and D will be zero in the circuit shown in Fig. 2.32 @ 42 i) 22 (iii) 3 (iv) 62 42. Acell having an e.m-f. of 2-2 V and internal resistance 0-2 Q is connected to a circuit comprising an ammeter and aresistance of 4 Q in series with a combination of two resistances of 0-4 Q each in parallel. What will be the reading of the ammeter ? 22V,020 1 042 4a o4n 3 OSA (i) ISA (ii) LSA (iv) LA 43, Two batteries of different e.m.fs and same intemal resistances are connected in series with an external resistance and current is 3 A. ‘When polarity of one is reversed, the current is 1 A. The ratio of e.m.fs is (i) 25 (ii) 2 (ii) VS (jt 44. Five resistances are connected as shown in Fig. 2.34. The effective resistance between points A and Bis Fig. 234 () 0B Q (ii) 2023 Q (i) 15 Q (@) 62 45, n similar resistors cach of resistance r when connected in parallel have the total resistance R. When these resistances are connected in series, the total resistance is (aR (i) Rin? (if) n?R (iv) Rin 46. Kirchhoff’s current law at a junction deals with (i) conservation of energy Gi) conservation of momentum Gif) conservation of angular momentum Giv) conservation of charge 47, Fig. 2.35 shows part of a closed circuit. What is the value of V,- V,? 2a 22 3V 12 ae — | wi 08 Fig, 2.35 ( 12V (i) 9V (iii) BV (iv) 6V 48. Kirchhoff's voltage law deals with (i) conservation of energy (ii) conservation of charge Objective Electrical Technology (iii) conservation of momentum (iv) conservation of angular momentum 49. A wire of resistance 0-1 Q/cmis bent to form a square ABCD of side 10 cm. A similar wire is connected between B and D to form the diagonal BD. If a 2 V battery of negligible intemal resistance is connected between and C, then total power dissipated is @2W (W3W (i) 4W (iv) OW 50. A cell supplies a current of 0-9 A through 2.Q extemal resistance and 0-3 A through 7 Q external resistance. The internal resistance of the cell is @ 059 (i) 12 Gi) 1Q (iv) 15Q 51. Four resistances each of value 4 Q are connected as shown in Fig. 2.36. The equivalent resistance between points A and Bis 42 4Q 42 A 42 8 Fig, 2.36 @ 32 (ii) 6Q (iii) 9Q (iv) 129 52, The resistance between P and Q in the circuit shown in Fig. 2.37 is Fig. 2.37 () 6Q (ii) 12 (iii) 18 Q (iv) 242 D.C. Circuits 53. Eight resistors each of resistance 10 Q are connected as shown in Fig. 2.38. The resistance between points A and B is wa we wa A 102 10 92 9 1092 Fig. 2.38 @ 302 (i) 602 (iii) 45. (iv) 90.2 54, A 50 V battery is connected across 10 © resistor. The current is 4-5 A, The internal resistance of the battery is © wero (i) 52 (iii) 05Q (ivy) 112 55, What is the equivalent resistance between the terminals A and B in Fig. 2.39 c R R o R Fig. 239 () 2R (i) 3R (iii) R (iv) RB 56, It is known that the potential difference across 6 Q resistor in Fig. 2.40 is 48 V. The entering current / is a+ 39 (3A (i) OA (ii) 9A (iv) IZA 57. In Fig. 2.40, the potential difference across 8 Q resistor is ( 48v (ii) 24V (ii) 960 V (iv) 96 V 58, Eight resistances each of resistance 10 Qare connected as shown in Fig. 2.41, The resistance between points A and B in the circuit is 102 102 102 A 102 1909 109 B Fig. 2.41 @ 30Q (ii) 60Q iii) 45Q (iv) 90Q 59. Fig. 2.42 shows a network of resistances. The effective resistance between points A and B of the network is 3% 99 pq 32 A sa 8 Fig. 2.42 () 32 (i) 20 (ii) SQ (iv) 69 60. In the circuit shown in Fig. 2.43, find the potential difference V,~ Vg 6Q 6Q6Q gs 6a \ PN go 4 4 Fig. 2.43 (i) 60V @) 72V @36V Gi) 30V 40 61. In the circuit shown in Fig. 2.44, the galyanometer reads zero. The value of resistance R is Fig. 2.44 (@ 22 (i) BQ (iii) 21Q (vy) MQ 62. Five resistances are connected as shown in Fig. 2.45. The equivalent resistance between points A and B is D 72 c Lb K B 102 Fig. 2.45 @ 10Q (ii) DQ (iii) 32 (is) HQ 63. The equivalent resistance of the arrangement of resistances shown in Fig. 2.46 between points A and B is 8a @ 62 Gi) 8Q (itt) 16 (i) 4Q 64. An ordinary dry cell can delivera current of about @IA Gi) 2A (ii) VBA (iv) none of above Objective Electrical Technology 65. Four cells, each of internal resistance | Q, are connected in parallel, The battery resistance will be @ 4Q (ii) 0252 (ii) 22 (wv) 12 66. Inthe circuit shown in Fig. 2.47, the reading of ammeteris (internal resistance of battery is zero) @ 10V 50 Fig. 2.47 @ 40294 Gi) 109A (iii) SBA (iv) 2A 67. In the circuit shown in Fig. 2.48, find the potential difference across cells E, and E; E=4V &=BV KS 052 19 3a 450 6Q Fig. 2.48 @ 375V,75V (ii) 425, 75V (iii) 375V,375V (iv) 425V,425V 68, Three resistances of 42, 6 Q and 10 © are connected in parallel ina circuit with a battery em. of 4°53 V. [See Fig. 2.49] If current through 6 Q resistance is 0: 6 A, the internal resistance of the battery is 42 Fig. 2.49 (i) 032 @) 05Q @ 02a (iii) 049 D.C. Circuits | 69. Referring to Fig. 2.50, the resistance across terminals AE is * 139 8 119 c wer 5a F E 92 D Fig. 2.50 @9Q Gi) 18Q Gi) NQ (iv) none of above 70. A battery of e.m.f. 2 V and internal resis- tance 0-5 Qs connected to a circuit consist- ing of a resistance of 5 Q and a galvanom- eter of resistance 15 & connected in series (See Fig. 2.51). Whena resistance of 3 Qis connected across the terminals of the galva- nometer, what is the current through the galvanometer ? 2v —— 05a ' 4 5a 1p BZ 3a Fig. 2.51 () 0044 (i) 1404 Gil) 040A (iv) 41A 71. In the circuit shown in Fig. 2.52, the resistance R is increased. What will be the effect on the galvanometer reading if the internal resistance of the battery is zero? {| Fig. 252 (i) it will not change Gi) it will decrease an (iii) it will increase (iv) data is insufficient 72. Inthe above question, ifthe battery has finite value of internal resistance, then what will be the effect on galvanometer reading ? (@ it will not change (ii) it will increase (iii) it will decrease (iv) data is insufficient 73. The current in a coil of resistance 90 &2 is to be reduced by 90%. What value of resistance should be connected in parailel with it? @g9Q i) 100Q Gi) 9Q (iv) 100 74. Ifa battery of 6 V is applied across terminals 1 and 2 in Fig. 2.53, then current in the horizontal 2 Q resistor will be 1Q 10 19 2a Fig. 2.53 @1A (i) 2A (it) 3.4 (iv) OSA 75. The potential difference between points A and B in Fig. 2.54 will be 5Q §Q A B 52 ? gy 5Q 5Q 5Q Fig. 2.54 @ BY (i) 3V (iii) 89 V (iv) 43V 76. In order to get maximum current in series- parallel grouping of cells, the external resistance should be .. the total internal resistance of the battery. 42 (i) less than (ii) more than (iii) equal to (iv) none of above 77. Two equal resistances are connected in series across a certain supply. If the resistances are now connected in parallel, the power produced will become (i) twotimes (ii) four times (iii) one-half (iv) none of above 78. In Fig. 2.55, the switches S, and S, are closed. Then total circuit resistance will be 100, 2007 40 W, 200V Le uy 100 W, 200V 200 5. Fig, 2.55 ( 4002 (i) 1200 Q (iii) 1000 9 (i) 2400 Q 79. In Fig. 2.55, both switches S, and S, are closed, Then (i) L, will be brighter than L, or Ly (ii) L, will be dimmer than Ly or Ly (iii) L, will be as bright as L, or Ly (iv) none of the above 80. In Fig. 2.55, switches S, and Sy are closed and the supply voltage is increased to 400 V. Then, (i) lamp L, will burn out (ii) lamp L, will burn out (iti) both lamps L, and L, will burn out (iv) all the lamps will be safe 81. In the circuit shown in Fig. 2.56, a battery of 10V tht 1y t 5002 5002 ore 1000 2 Fig. 2.56 Objective Electrical Technology e.m.f, 10 V andnegligible internal resistance is connected across two resistances of 500 Q in series. A voltmeter of 1000 Q Tesistance is connected across one resistance. What is the reading of the voltmeter ? @4Vv (ii) 2V (iii) 3V ()1V 82. Voltmeters V, and V, are connected in series across a d.c. line. The voltmeter V, reads 80 V and has per volt resistance of 200 Q while V, has a total resistance of 32 kQ. The line voltage is @® 120V (i) 160 V (iii) 2OV (iv) 240 83, Fig. 2.57 shows a part of a closed circuit. ‘The potential difference between points A and Bis BV 6v 3a_ 82 39 ow—_ Wi t Fig, 2.57 @ 12V (ii) 24V (iii) 18.V (iv) 29V 84, In the Wheatstone bridge shown in Fig. 2.58, P=99;Q=110,R=4Q and $=6. How much resistance must be put in parallel to resistance S to balance the bridge ? Fig, 2.58 242 (ii) 2642 (iii) 15 () 12 85. A galvanometer together with an unknown resistance in series is connected across two identical batteries each of 1-5 V. When the batteries are connected in series, the galvanometer records a current of | A and D.C. Circuits when the batteries are connected in parallel, the current is 0:6 A. What is the internal resistance of each battery ? 12 (ii) 05Q (i) 13.2 (iv) 15.0 86. Six equal resistances, each of 2 Q, are connected as shown in Fig. 2.59. The resistance between any two comers is, A 22 8 Fig. 259 12 (i) 42 (iii) 22 (iv) 82 87, Resistances 1 @,2Q and 3 Q are connected in the form of a triangle. If a cell of em. 15 V and negligible internal resistance is connected across 3 Q, the current through this resistor is (i) 025.4 i) OSA (iii) LSA (iv) 25A 88. The current in 2 Q resistor shown in Fig. 2.60 is Fig. 2.60 (ii) O4A @12A (ii) b4 A () 1A 89. A wire has a resistance of 6 ohms. It is bent in the form of an equilateral triangle. The effective resistance between any twocomers of the triangle is (472 (i) 6Q (iii) 473. Q (iv) 32. 90. A charge of 10 mC flows through a cross- 43 section 2:5 mm’ normally in 2 seconds, The current is () 25mA (i) SMA (iii) 20mA tiv) 4mA 91. Two similar cells whether joined in series or in parallel have the same current through an external resistance of 2 &. The internal resistance of cach cell is (22 (i) 12 (ii) O5 Q (vy) 152 92. When resistances are connected in parallel, the current divides itself in (i) direct ratio of resistances (ii) inverse ratio of resistances (iii) inverse ratio of potentials (iv) none of above 93. When a d.c. battery of e.m.f. E and internal resistance r delivers maximum power to an external resistance R, the ratio r/R is @a:t (i) 2:1 (iii) 1:2 (i) 1:12 94, Five identical lamps each of resistance R = 1100 Q are connected to a 220 V supply as shown in Fig. 2.61. The reading of ideal ammeter is t PBR , Fig, 2.61 @ USA (ii) USA Gi) ISA (iv) LA 95. A torch bulb rated at 4-5W, 1-5V is connected as shown in Fig. 2.62. The e.m.f. of the cell iv needed to make the bulb glow at full intensity is hs 12 @45v Gi) 15 * —_— (iii) 256 V E,r=267Q Gv) BSV Fig. 2.62 44 96. ‘Two bulbs rated at 25 W, 110 V and 100 W, 110 V are connected in series to a 220 V supply. What will happen to the circuit ? (i) 100 W bulb will burn out (ii) 25 W bulb will burn out Gif) both bulbs will burn out (iv) no bulb will burn out 97. Three equal resistors connected in series across a source of e.m.f. together dissipate 10 W. What would be the power dissipated if the same resistors are connected in parallel Objective Electrical Technology 99. A standard 40 W tube light is in parallel with room heater and both are connected to main supply line. What will happen when light is switched off ? (i the heater output will increase (ii) the heater output will decrease (ii) the heater output will remain same (iv) none of above 100. Twelve wires, each of resistance R, are connected to form a cube. The effective resistance between two diagonal ends (points across the same source of e.m.f. ? 1 and 7 in Fig. 2.63) will be () 90W (i) 30W 7 (iii) 180 W (iv) 27W 98, In acircuit, two cells of 1-5 Vand 2V em. 2 having internal resistances of 1 Q and 2 Q respectively are connected in parallel so as to send currentin the same direction through s an extemal resistance of 5 Q. The circuit currentis t 4 @ Za wa Fig. 2.63 5 5 SR .. OR Gi) 4 ww Sa Os ws 7 2 (id) 3R (iv) 12R ANSWERS TO OBJECTIVE QUESTIONS L@ 2, (ii) 3. (iii) 6. (ii) 1. (ii) 8 M. (i) 12. (iv) 13. (iii) 16, (iv) 17. (ii) 18, (ii) 21. (iii) 22. (iv) 26. (iv) 21, (ii) 31. Gi) 32. (iv) 3%. 37. (iv) 41. wv) 42. 46. (iv) 47. (ii) 51. 52. (ii) 56. (iv) 57. (iv) 61. (i) 62. (iii) . (Hi) 66. (iv) 67. (ii) 69. (iii) 71. (i) 72. (ii) 74. (iii) 76. (iii) 77. (ii) Di) 81. 82. (iv) 84. Gi 86. Gi) 87. (ii) 89. (iii) D.C. Circuits 45 91. 92. (ii) 93. 94, (iii) 95. (iv) 96. (ii) 97. (i) 98. (iii) 99. (iii) 100. (i) 10. uu. 18. 24, HINTS TO SELECTED OBJECTIVE QUESTIONS ‘The resistance of 40 W lamp is much more (2-5 times) than that of 100 W lamp. Consequently, a greater part of supply voltage will appear across 40 W lamp. So far as terminals AB are concemed, 12 Q and 6 Q are in series and this series combination is in parallel with 18 Q resistor. R= (2+Ox18 _ 18x18 _ 2 Se =92 (2+ O4+18 36 Cireuitcurent, = 180220 Rag 9 ‘Since there are two parallel paths of equal resistance, current in each path is 10 A. 6x6 8x8 Rep= Gag 2325 Reo = Raa= gy g = 42 i ig PRO Circuit current = Rs 6A Current in 5Q Current in 2Q Fig. 2.64 shows the conditions of the problem. Each parallel path sa has aresistance of 6 Q. .. Effective resistance between two points on the diameter = wfX6. 36, 6a = 616= 676712792 Fig. 264 E___ 20 Circuit current, Te Rey 39401" Voltage across cell, V = E—Ir=20-0-5x0- Conductance G is the reciprocal of resistance R i.e. or Pr = Be ~ 200+100 46 29. 32. 3B 37. 3». 42. Objective Electrical Technology It may be seen that one end of each resistor is connected to point A and the other end of each resistor is connected to point B. Hence the three resistors are in parallel. a Rap or Rus = The effective resistance between points E and Fis (10 +10) x10 _ 200 _ 20 ‘er = (0410) +107 307 3° 20 80 Rye = 10+ 2+ 10- Bo In the steady state, the capacitor offers infinite resistance to direct current. Therefore no current flows in the branch containing r, and C. Asa result, the potential difference across C is that across ry. - Vv Curentthrough 7, = —Y— PD. across r, = {—V_) x, = V2 ith nth Let E and r be the e.m.f. and internal resistance of each cell. For series grouping, 2E = (5+2r)x 10 wali) For parallel grouping, E = (s + 5) x8 Ait) Solving eqs (i) and (ii), we get, r= 2-5 Q Applying Kirchhoff’s voltage law to the circuit, we have, V,+3-3x1-3x6 = V, or V,+3-3-18 = J, * -Vy = 18V Current in branch DAC as well as in branch DBC is | A. Vp-2X1 = Vy and Vy-3X1=Vy Vy-V_ = (Vy-2)-(Vp-3)5+1V When points B and D are at the same potential, this Wheatstone bridge is balanced. Under balanced conditions of the bridge, the products of resistances of opposite arms are equal. G+DX = «a+9x(25) I+] or xX =22 We consider the ammeter to be ideal so that its resistance is zero. -. Total circuit resistance is Circuit current, Therefore the reading of ammeter will be 0°5 A. Let E,, Ey be the e.m.is of the wo cells and rbe the internal resistance of each cell, If Ris the external resistance then, At 23 og S22 21 Retr Rr D.C. 45, 50. $2. 5S. 56, 61. 66. Circuits a7 Ste o3 B-% On solving, E/E, =2 Rerin x r=aR When n resistors, each of resistance r (= n R) are connected in series, then total resistance is Ro =nxnaRawR Let £ and rbe the e.m.f. and internal resistance of the cell respectively. For the first case, 09 = E. i) I+ For the secondewe, 03 = —£ Ai) T+r Solving eqs (@) and (ii), we get, r= 05 Q Here PROSP is a balanced Wheatstone bridge. Therefore branch RS is ineffective and can be considered as removed, 20 x 30 20 + 30 The network shown in Fig. 2.39 can be redrawn as shown in Fig. 2.65 (i). It is a balanced Wheatstone bridge. Therefore, point Cand D are at the same potential. Since no current flows in the branch CD, this branch is ineffective in determining the equivalent resistance between terminals A and B and can be removed. The circuit then reduces to that shown in Fig, 2.65 (ii). Rpg = 202|| 302= =12Q (ii) Fig. 2.65 The branch ACB (= R + & = 2R) is in parallel with branch ADB (= R + R= 2R), = (2R)x(2R) Rea = “oreor ~* Current through 6 Q resistor, 1, =48/6=8A Current through 12 Q resistor, /, =48/12=4 A Entering current, / = 1, +/,=8+4=12A Itis a balanced Wheatstone bridge. Therefore product of resistances of opposite arms is equal he. 40x7 = 10KR or r= ®* Log We consider the ammeter to be ideal so that its resistance is zero. 4x5 _ 20 Total circuit resist: = s— cuitresistance = 4%3 = 20 9 10 _90_9 Circuit current = 3575 = 39 = 3 70. 7B. 82. Objective Electrical Technology istor = 2 —4_ Current through 5 Q resistor = 3" Therefore, ammeter reading will be 2A. The equivalent resistance of parallel combination of 15 Q and 3 Q is R= 1543 Total circuitresistance, Rp = O05 +54+25=82 Total circuit current, I Current through galvanometer, J, The conditions of the problem are represented in Fig. 2.66. The voltage across the required resistance S and 90 Q is the same ie. O91xS = Ox 90 or S = 909= 100 Suppose each resistance is R. Forseriesconnection, P, = 2R ve For parallel connection, P; = p73 zB, 7 =4 or P,=4P, 1 The resistance of each 100 W lamp is 400 Q. The total resistance of the parallel circuit will be 200 2. The resistance of 40 W lamp is 1000 @. The applied voltage of 400 V will appear across 40 W lamp and the parallel circuit in the ratio of 1000 ; 200 = 5 : 1. Consequently the voltage across 40 W lamp will far exceed 200 V and it will burn, Resistance of parallel resistances of $00 Q and 1000 Q is 1000 x $00 _ 1000 R, = 000+5007 3 2 Total circuit resistance is By = 1004 500= 50 a Circuit pg eel et. t current, =e: = ircuit current, R, 250073 ~ 250 Current in 500 Q resistor across which voltmeter is connected is = fe 1000.3 1000 1 ! 300 +1000 ~ 250 “ 1500 ~ 125 PD. across 500 Q resistor = 1,x 500 Fag x S04 Therefore, the voltmeter will read 4 V. Resistance of voltmeter V, is R, = 80x 200 = 16000 Q= 16kQ . y%_ 80v 3 si a BOY = Circuit current, T= RA jgEgTSMA=SxIOPA Reading of V, = IR, =5 mA x 32kQ=160V Line voltage = V, + V, = 80+ 160 =240V OG. 87. 91, 95. 98. 100, Circuits Fig. 2.67 shows the conditions of the problem. i Total circuit resistance is (2+)x3 1 = G@epes 1 5e 15V EUS. Circuitcurrent, = J = Zs” * Current in 3 Q resistor is 3 a3" ix3=0- SA I =Ix 3, Fig, 2.67 Let £ and r be the e.m.f. and internal resistance of each cell respectively. 28 1 E 24% ~ 2er/ or 242 ~ 2¥r72 Onsolving, we get, rr = 20 2 Resistance of bulb, —-R, = © -43 =0:5Q P Rated curentofbulb, 1, = V3=3A In order that bulb glows at full intensity, 3 A must pass through it, Total circuit resistance, Rp = 2-67+ 9° 3%) 26740336 32 Current supplied by battery, / 1 pa X15 -F,1 or 328 31. or E = 3x3x15=135V Apply Kirchhoff’s voltage law to solve the problem. Symmetry about entrance point 1 and exit point 7 shows that points 2, 4 and 5 are at the same potential. Similarly points 3, 6 and 8 are at the same potential. Therefore the circuit reduces to the one shown in Fig, 2.68. a o 2 = 3 Network Theorems CHAPTER OVERVIEW 1, NETWORK Any arrangement of electrical energy sources, resistances and other circuit elements is called an electrical network. The terms circuit and network are used synonymously in electrical literature. Thus Fig. 3.1 may be referred to as an electrical circuit or network. 2. NETWORK TERMINOLOGY While discussing network theorems and techniques, one often comes across the following terms : () Active element. An active element is one which supplies electrical energy to the circuit. ‘Thus in Fig. 3.1, E, and E; are the active elements because they supply energy to the circuit. (i) Passive element. A passive element is one which receives electrical energy and then either converts it into heat (resistance) or stores in an electric field (capacitance) or magnetic field (inductance). In Fig. 3.1, there are three passive elements, namely R,, R, and R;. These passive elements (c.e., resistances in this case) receive energy from the active elements (i.e., E, and E,) and convert it into heat. (iii) Node. A node of a network is any rR, 8 Ry c equipotential surface at which two or more circuit elements are joined. Thus is Fig. 3.1, cifcuit elements R, and E, are joined at A and hence A is the node, Similarly B, C and D are nodes. (iv) Junction. A junction is that point in a network where three or more circuit elements are joined. In Fig. 3.1. there are only two junction Fig. 3.1 points viz.,B and D, That Bis a junction is clear from the fact that three circuit elements R,. R, and R, are joined at it. Similarly, point D is a junction because it joins three circuit elements R,, E, and Ey. (») Branch, A branch is that part of a network which lies between two junction points. ‘Thus referring to Fig. 3.1, there are a total of three branches viz., BAD, BCD and BD. The branch BAD consists of R, and E, ; the branch BCD consists of Ry and E, and branch BD merely consists of Ry (vi) Loop. A loop is any closed path of a network. Thus in Fig. 3.1, ABDA, BCDB and ABCDA are the loops. (vi) Mesh. A mesh is the most elementary form of a loop and cannot be further divided into other loops. In Fig. 3.1, both loops ABDA and BCDB qualify as meshes because they cannot be further divided into other loops. However, the loop ABCDA cannot be called a "sh because it encloses two loops ABDA and BCDB. 50 0 Network Theorems 51 3. IDEAL AND REAL VOLTAGE SOURCES It is very important to understand ideal and real voltage sources. (i) (ii) Ideal Voltage source, An ideal voltage source is one that maintains a constant terminal voltage no matter how much current is drawn from it. For example, an ideal 10 V source would theoretically maintain 10 V across its terminals when a 1 M Q resistor is connected (so /= 10 V/I MQ = 10 pt A) as well as when a I k Q resistor is connected (/= 10 m A) or when a | Q resistor is connected (J = 10 A) and even when a 0.01 @ resistor is connected {= 1000 A). Note that an ideal voltage source has constant terminal voltage regardless of current supplied by it. Clearly, an ideal voltage source has zero internal resistance. Fig. 3.2 (i) shows a 10 V ideal voltage source. Tdeal Source Real Source @ ww Fig 32 Real Voltage source. There is no such thing as an ideal voltage source. A real source has internal resistance R, that acts in series as shown in Fig. 3.2 (ii). ‘The internal zesistance causes the terminal voltage to drop if the current is made sufficiently large (i.e., if a small enough resistance is connected across the terminals). As the intemal resistance becomes smaller, the real voltage source more closely approaches the ideal source. In many situations, itis convenient to assume a real voltage source like an ideal one. 4. IDEAL AND REAL CURRENT SOURCES We now discuss ideal and real current sources. @ i) Ideal Current source. An ideal current source (also called constant-current source) will supply the same current to any resistance connected to its terminals. Fig. 3.3 (i) shows | A ideal current source. This source supplies 1 A current to load R, whether R, = t Q or 10 Q or 100 Q and so on, Fig. 3.3 (Zi) shows the symbol used to represent an ideal current source, Note RL that symbol fora current source is a circle with an arrow. The arrow shows the direction of the (conventional) current produced by the source. @ Since an ideal current source supplies the same (i) current to any load resistance, it is clear that the Fig. 3.3 voltage across the terminals of the source must change if load resistance (R,) changes. Thus in Fig. 3.3 (i) if R, = 10 Q, thea terminal voltage F=7R, =| Ax 10Q = 10 V. If resistance is changed to 100 Q, then E = 1 x 100 = 100 V, Real Current source. There is no such thing as ideal current source. It is because a real current source always has some internal resistance that causes current io drop if the voltage developed across the terminals become sufficiently large (i.e., the resistance connected across 52 Objective Electrical Technology the terminals is made too large). Areal current source ~------------~ has internal resistance R, that acts in parallel with the ! source, The value of this shunt resistance Ry | determines how closely the current source approaches | the ideal one. The larger the value of Rg, the more {1 at) PR closely the current source approaches ideal one. Note | that real current source is a converse of real voltage | source where Rg acts in series and must be small for !___ _"~=-__—=7_2 the source to become constant voltage source. 5, NETWORK THEOREMS AND TECHNIQUES There are certain circuits that cannot be solved using Ohm's law and Kirchhoff’s laws. To overcome this difficulty, other network theorems and techniques have been developed which are very useful in analysing both simple and complex electrical circuits. Through the use of these theorems and techniques, it is possible either to simplify the network or render the analytic solution easy. The network theorem or technique to be used depends upon the network arrangement, The general rule is this. Use that theorem or technique which requires a smaller number of independent equations to obtain the solution or which yields easy solution. 6. MAXWELL’S MESH CURRENT METHOD In this method, Kirchhoff’s voltage law is applied to a network to write mesh equations in terms of ‘mesh currents instead of branch currents. Each mesh is assigned a separate mesh current. This mesh current is assumed to flow clockwise around the perimeter of the mesh without splitting at a junction into branch currents. Kirchhoff’s voltage law is then applied to write equations in terms of unknown mesh currents. The branch currents are then found by taking the algebraic sumof the mesh currents which are common to that branch. Explanation. Maxwell's mesh current method consists of following steps : (© Each mesh is assigned a separate mesh current, For convenience, all mesh currents are assumed to flow in *clockwise direction. For example, in Fig. 3.5, meshes ABDA and BCDB have been assigned mesh current /, and /, respectively. (i) If two mesh currents are flowing through a circuit element, the actual current in the circuit elements is the algebraic sum of the two. Thus in Fig. 3.5, there are two mesh currents /, and 1 flowing in Ry. If we go from B to D, current is 1, — 1 and if we go in the other direction (i.e., from D to B), current is I, Real Current Source Fig. 3.4 Fig. 3.5 (iii) Kirchhoff’s voltage law is applied to write equation for each mesh in terms of mesh currents. * Ibis convenient to consider all mesh currents in one direction (clockwise or anticlockwise). The same result will be obtained if mesh currents are given arbitrary directions, Network Theorems 53 Remember, while writing mesh equations, rise in potential is assigned positive sign and fall in potential negative sign. (iv) If the value of any mesh current comes out to be negative in the solution, it means that true direction of that mesh current is anticlockwise i.¢., opposite to the assumed clockwise direction. Applying Kirchhoff's voltage law to Fig. 3.5, we have, Mesh ABDA -1R,-(,-h) +E, = 0 or 1,(R, +R)- LR, = E, ai) Mesh BCDB ~ LR, -E,-(,-1)R, = 0 or = 1,R, + (Ry +Ry) b= -E, waif) Solving eq (i) and eg (ii) simultaneously, mesh current /, and I, can be found out. Once the mesh currents are known, the branch currents can be readily obtained. The advantage of this method is that it usually reduces the number of equations to solve a network problem. 7. NODAL ANALYSIS Consider the circuit shown in Fig. 3.6, The branch currents in the circuit can be found by Kirchhoff’s laws or Maxwell’s mesh current method, There is another method, called nodal analysis, for determining the branch currents in the circuit. In this method, one of the nodes (Remember a node is a point in a network where two or more circuit elements meet) is taken as the reference node. The potentials of all the points in the circuit are measured w.r:tthis reference node. In Fig. 3.6, A, B, Cand Dare four nodes and node D has been taken as the "reference node. A glance at the circuit shows that voltages at nodes A and C wirt, reference node D are known. These are £, = 120 V and E, =65 V respectively. The only potential of node B w.ri. D (call it Vg) is unknown. If this potential Vy can be found, each branch current can be determined because the voltage across each resistor will then be known. Hence nodal analysis essentially aims at choosing a reference node in the network and then ‘finding the unknown voltages at the nodes w.rt. reference node. « R, i og ok Ry c E,=120V Vo =r, E,=65V Reference Node Fig. 3.6 The voltage V, can be found by applying Kirchhoff's current law at point B. hth =k efi) In mesh ABDA, the voltage drop across R, is E, — Vp. Ke z 7 Vp — i An obvious choice would be ground or common, if such 2 point exists. 54 Objective Electrical Technology In mesh CBDC. the voltage drop across R, is E, ~ Vz. Also current I, = V,/R). Putting the values of /,, J, and /, in eq (i), we get, 4a, BoM tii) RX x Rg Alll quantities except Vg are known, Hence Vz can be found out. Once Vy is known, all branch currents can be calculated. It may be seen that nodal analysis requires only one equation (eq, (ii)] for determining the branch currents in this circuit. However, Kirchhoff’s or Maxwell’s solution would have needed two equations. 8. SUPERPOSITION THEOREM ‘Superposition is a general principle that forms the basis fora very powerful technique used to analyse multisource electric circuits. The essence of superposition theorem is that an e.m.f. acting in a “linear network produces the same effect whether it acts alone or in conjunction with other e.m.f.s. This theorem may be stated as under : Ina linear network containing more than one source of emf, the resultant current in any branch is the algebraic sum of currents that would be produced by each e.m{. acting alone, all other sources of emf. being replaced meanwhile by their respective internal resistances. The superposition theorem can also be used to determine voltage across any component of a multisource electric circuit. To determine the current through or voltage across any component of a multisource circuit using superposition theorem, we first compute the current or voltage due to each source acting alone. Such computation requires that we remove all sources except one. Sources are removed by making their contributions equal to zero i.e. we (i Replace ideal voltage sources by short circuit (E = 0). (ii) Replace ideal current sources by open circuit (I = 0). Note that these steps are equivalent to replace real sources by their internal resistances. Explanation. To show that this statement 1 R, is true, consider the circuit shown in Fig. 3.7. The total circuit resistance Ry is given by : Ry =R, + Rp Ry RR where Rp = PO RAR, Circuitcurrent, 7 = Netem.f. Let us now find the cicéuit current by superposition theorem. Consider that e.m.f. E, is acting alone by replacing E, with a short** circuit between C and D. * — Allinear circuit is one whose parameters (e.g. resistances) are constant i.e. they do not change with current or voltage. This theorem is only restricted to linear network. ** Because it is assumed that E, and E, have zero intemal resistances. Network Theorems 35 —_ A RR, +R, Now replace F, witha short circuit between A and E so that now E; is acting alone. Current due to E, alone, f, = Current due to E, alone, /,= Re + Ret Ry The minus sign with /, indicates that current produced by E; alone is in a direction opposite to that produced by E, alone. Circuit current, I Algebraic sum of /, and /, 5 .{_25 Rt Rpt R,) (R+RptR, jp ee R+R +R, R+Rp+R, This establishes the validity of superposition theorem. which is same as eq. (i). 9, THEVENIN'S THEOREM Fig. 3.8 (i) shows a network enclosed in a box with two terminals A and B brought out. The network in the box may consist of any number of resistors and e.m.f, sources connected in any manner. But according to Thevenin, the entire circuit behind terminals A and B can be replaced by a single source ofe.m.. Ep, (called Thevenin voltage) in series with a singe resistance R,, (called Thevenin resistance) as shown in Fig. 3.8 (ii). The values of Ey, and Ry, are determined as mentioned in Thevenin’s theorem. Once Thevenin’s equivalent circuit is obtained (See Fig. 3.8 (ii)], then current through any load resistance R, connected across AB is given by : COMPLEX CIRCUIT @ Fig 38 i) Hence Thevenin’s theorem as applied to d.c. circuits may be stated as under : Any neiwork having terminals A and B can be replaced by a single source of e.mf. Ey, in series with a single resistance Rr. (i) The emf. Ey, is the voltage obtained across terminals A and B with load, if any, removed ie, itis the open-circuited voltage between A and B. (ii) The resistance Ry, is the resistance of the network measured between A and B with load removed and sources of e.m.f, replaced by their internal resistances. Tlustration, Consider the circuit shown in Fig. 3.9 (7). As far as the circuit behind terminals AB is concerned, it can be replaced by a single source of e.m.f. E,, in series with a single resistance Ry, as shown in Fig. 3.9 (ii). Thee.m.f. E>, is the voltage across terminals AB with R, removed. With R, disconnected, there is no current in R, and Ey, will be voltage appearing across R;. 56 Objective Electrical Technology Voltage across Ry Current through R, x Resistance R, v = ><, R+R > Ry Fe Be gt w@ (ii) Fig. 39 To find Ry, remove the load R, and replace the battery by a short-circuit because its internal resistance is assumed zero. Then resistance measured between A and B is equal to R,, Obviously, looking back into the terminals AB, R, and R, are in parallel and this parallel combination is in series with R,. R R = Rt Fn = BRR, When load R, is connected between terminals A and B, then current in R, is given by : fz 10. NORTON’S THEOREM Fig. 3.10 (i) shows anetwork enclosed ina box with two terminals A and B brought out. The network in the box may contain any number of resistors and e.m.f. sources connected in any manner. But according to Norton, the entire circuit behind AB can be replaced by a current source J, in parallel with a resistance Ry as shown in Fig. 3.10 (ii). The resistance Ry is the same as Thevenin resistance Rp, The value of /y is determined as mentioned in Norton's theorem. Once Norton's equivalent circuitis determined {See Fig. 3.10 (éi)], then current through any load R, connected across AB can be readily obtained. A }+——+ COMPLEX a NETWORK 4 +. 8 @ Fig. 3.10 Hence Norton's theorem as applied to d.c. circuits may be stated as under : Any network having two terminals A and B can be replaced by a current source of current output Jy in parallel with a resistance Ry. {The output Ly of the current source is equal to the current that would flow through AB when A and B are short-circuited. Network Theorems 57 (ii) The resistance Ry, is the resistance of the network measured between A and B with load removed and the sources of e.m.f., replaced by their internal resistances. Norton's theorem is converse of Thevenin’s theorem in the following respect. Norton equivalent circuit uses a current generator instead of voltage generator and the resistance Ry (which is the same R,,) in parallel with the generator instead of being in series with it. Illustration. Fig. 3.11 illustrates the application of Norton's theorem. As far as the circuit behind terminals AB is concerned {see Fig. 3.11 (i)], it can be replaced by a current source J, in parallel with a resistance Ry as shown in Fig. 3.11 (iv), The output fy of the current generator is equal to the current that would flow when terminals A and B are short circuited as shown in Fig. 3.11 (ii). The load on the source when terminals AB are short-circuited is given by : R = 3+ eB. RETREAT RR R+R, R+R ov V(R, +R) Source current, rs = ae RRR, +RR+ RR Short-cireuit current, Jy = Current in R, in Fig. 3.11 (ii) 6 1 Be RtR RRARR+RR az tf: (iii) ty) Fig. 3.11 To find Ry, remove the load R, and replace battery by a short because its internal resistance is assumed to be zero [see Fig. 3.11 (li). = Resistance at terminals AB in Fig. 3.11 (iii) RR = R+ BRR Thus the values of Ly and Ry are known. The Norton equivalent circuit will be as shown in Fig. 3.11 (iv). 11. MAXIMUM POWER TRANSFER THEOREM This theorem deals with transfer of maximum power from a source to load and may be stated as under : 58 Objective Electrical Technology In de. circuits, maximum power is transferred from a source to load when the load resistance is made equal to the internal resistance of the source as viewed from the load terminals with load removed and all emf. sources replaced by their internal resistance: CIRCUIT R, @ Fig. 3.12 Fig. 3.12 (i) shows a circuit supplying power to aload R,.. The circuit enclosed in the box can be replaced by Thevenin’s equivalent circuit consisting of Thevenin voltage E (= E,) in series with ‘Thevenin resistance R, (=R,,) as shown in Fig. 3.12(ii). Clearly resistance , is the resistance measured between terminals AB with R, removed and e.m.f, sources replaced by their intemal resistances. According to maximum power transfer theorem, maximum power will be transferred from the circuit to the load when R, is made equal to R,, the Thevenin resistance at terminals AB. The proof of this theorem is left as an exercise for the reader. Note. Under the conditions of maximum power transfer, the efficiency is only 50% as one-half of the total power generated is dissipated in the internal resistance R, of the source. 12. DELTA/STAR AND STAR/DELTA TRANSFORMATION There are some networks in which the resistances are neither in series nor in parallel. A familiar case is a three terminal network e.g. delta network or star network. In such situations it is not possible to simplify the network by series and parallel circuit rules. However, converting delta network into star and vice-versa often simplifies the network and makes it possible to apply series-parallel circuit techniques. Delta / Star Transformation. Consider three resistors Ryy, Rgc and Rc, connected in delta to three terminals A, B and Cas shown in Fig. 3.13 (i). Itis desired to replace these three delta-connected resistors by three resistors Ry, Ryand R-connected in star [See Fig. 3.13 (éi)] so that the two networks are electrically equivalent, The two arrangements will be electrically equivalent if resistance between any two terminals of one network is equal to resistance between the corresponding terminals of the other network. 4 A (ii) Network Theorems Referring to Delta network shown in Fig. 3.13 (i), Resistance between A and B= Ryg || (Ryo+ Rew) — Ryp(Rac+ Rea) Raat Rac + Rea Referring to star network shown in Fig. 3.13 (ii), Resistance between 4 and B= Ry + Ry Since the two arrangement are electrically equivalent, — Raa Bact Rea) Rath = Be Ryct Rex Similarly it can be shown that between terminals B and C and terminals C and A, Rac (Rea * Rup) Rat Ro = eee’ Raat Rac Bea Rpt R, and Ret R, = RealRen* Bac) Rant Root Rey Subtracting eq, (iv) from eq, (iii) and adding the result to eq. (v), we have, R, = <—RawRea__ 4 Raat Bact Rea R,-R, Similarly, Ry = > * 8 Rant Root Rea R, and R, = —— hea Rant Bact Bea How remember ? There is an easy way to remember it, Referring to Fig. 3.14, star-connected resistances Ry, Ry and Rare electrically equivalent to delta connected resistances Ray. Rye and Rey. We have seen above that : RapRea Rug + Rac + Rea ie. Any arm of star connection _ Product of two adjacent arms of A ~ ‘Sum of arms of A Star/delta Transformation, Now let us consider how to replace star-connected network of Fig. 3.13 (ii) by the equivalent delta connected network shown in Fig. 3.13 (i). ‘This can be easily done by considering eqs. (vi), (vii) and (viii) above. Dividing eq. (vi) by eq. (vii), we have, Ry = RyRy = Reg Roc Dividing eq. (v#) by eq. (viii), we have, RURe = RaglRoe c Ryg= ABE .. Gi) a. ii) (iv) we &) 60 Objective Electrical Technology Substituting the values of R,., and Ry, in eq. (vi), we have, _ [tte] ete) R A Ry Rac + Rac + pi Rac 2 se ae CR i (Ry P Re) +14 (Ry Ra) BE 2 a ee RyRy + ROR, + RARe or gg = Pakert RR + Rake Ry Rac = Ry Rot Be a Similarly, Rea = seem Ry R,R, and Ruy = Rat Rpt How to remember ? There is an easy way to remember it. Referring to Fig. 3.15, star-connected resistances R,, Ry and Re are electrically equivalent to delta connected resistances Ryy, Rec and Rey. We have seen above that : RAR, Rag = Ryt Ry tp ie. Resistance between two terminals of delta = Sum of star resistances connected to those terminals plus product of the same two resistances divided by the third star resistance. 13, RECIPROCITY THEOREM This theorem permits us to transfer source from one position in the circuit to another and may be stated as under : . In any linear bilateral network, if an e.mf. E acting ina branch X causes a current I in branch ¥, then the same e.m,. E located in branch ¥ will cause a current I in branch X. Fig. 3.15 A200 Bo A 2%2 ¢ 82 ED @| woo | + D E=100V wi) Fig, 3.16 Explanation. Consider the circuit shown in Fig, 3.16 (i). The e.m.f. E (=100 V) acting in the branch FAC produces a current amperes in branch CDF and is indicated by the ammeter. According Network Theorems 61 to reciprocity theorem, if the e.m.f. E and ammeter are interchanged* as shown in Fig. 3.16 (ii), then the ammeter reading does not change i.e. the ammeter now connected in branch FAC will read / amperes. In fact, the essence of this theorem is that E and are interchangeable. The ratio E/J is constant and is called iransfer resistance (or impedance in case of a.c. system). Qi Ans, Q2 Qo Ans. Qd Qs. Ans. Q6 Q7 SHORT ANSWER QUESTIONS Why do we use network theorems and techniques to solve electrical circuits. Network theorems and techniques are often used to solve electrical circuits for the following reasons : () Through their use, a complicated network can be reduced to a simple circuit. (ii) Through their use, the analytical solution becomes very easy. What is the proper use of the terms network and circuit ? ;» The term network is generally used in reference to an arbitrary arrangement of passive components while circuit usually implies the presence of active sources and the current flow, However, there is no hard and fast rule for making these distinctions and the terms “network” and “circuit” are often used interchangeably. What is the difference between a mesh and a loop ? A loop is any closed path of a network. However, a mesh is the most elementary form of a loop and cannot be further divided into other loops. What do you mean by a linear circuit element ? Circuit elements (e.g. resistance etc.) are linear when they follow Ohm’s law so that doubling the voltage across them doubles the current through them. What do you mean by a bilateral circuit element ? Circuit clements are bilateral if current can pass through them in either direction with equal ease What do you mean by an ideal voltage source and ideal current source ? An ideal voltage source is one that maintains a constant terminal voltage no matter how much current is drawn from it. For example and ideal 12 V voltage source theoretically maintains 12 V across its terminals whether you connect 1 MQ or 0-01 Q resistor across its terminals. An ideal current source is one which will supply the same current to any resistance connected to its terminals. For example a 2 A ideal current source will always supply 2 A to any value of resistance connected across its terminals. You are given two black boxes A and B shown in Fig. 3.17 and told that each contains a source of some type. Can you determine what is in each box ? Box A Box B 102 Sov @1a Fa @ (ii) Fig. 3.17 * Tfthe source of em.t. in the original circuit has an internal resistance, this resistance must remain in the original branch and cannot be transferred to the new location of the e.m.f. 62 Objective Electrical Technology Ans. (i) Since there is no way to see inside the boxes, a load is connected to each to determine what it contains, Suppose we connect a load of 10.2 to each box as shown in Fig. 3.18. Voltmeter and ammeter are also connected to obtain data. In case of box A, the 102 Joad resistor acts in series with 10 Q internal resistance so that total resistance is 20 2. Ammeter reading = 32 =05A Voltmeter leading = 10x05=5¥ Box A Box B 10a =10v 102 Din Zo o- @ (i) Fig. 3.18 In case of box B, two 10 @ resistors are in parallcl and the current of 1 A is divided equally between these resistors. Ammeter reading = 0-5A Voltmeter reading = 05x 10=5V ‘The meter readings are the same for two boxes so no difference is found as yet. (ii) Ifthe external 10 loads are removed, both voltmeters indicates 10 V and both ammeters read zero, (ii). If the external 10 Q loads are shorted, both ammeters read | A and both voltmeters read 0. Therefore, the boxes appear to be identical. The voltage source and current source of Fig. 3.17 are considered duals. Q.8. The voltage source and current source of Fig. 3.17 are called duals. Why is word dual used? Ans, The word dual is used because the voltage source and current source in Fig. 3.17 are not exactly equal. A black boxes analysis shows them to be so, but there are internal differences. For example, if both boxes are open-circuited, the voltage source delivers no energy but the current source supplies 1 A to its internal shunt resistance. Power is dissipated in the shunt resistance of current source under open circuit conditions. Q.9. How isa real (non-ideal) current source represented ? Ans. A real (non-ideal) current source can be represented as an ideal current source in parallel with internal resistance (R,,,) of the source shown in Fig. 3.19. When load R, is connected across the terminals , the current produced by the source jggqy divides between R,,, and R,, Consequently, the Current load current (/,) is less than it would be if the ‘Source source were ideal. Note that an ideal current source would have infinite internal resistance (i.e. Ri, in Fig. 3.19 would be replaced by an open circuit), In that case, all of the source current would be delivered to the load. Fig. 3.19 I Current Source Network Theorems 63 Q.10. How is a real (non-ideal) voltage source represented ? Ans. A real (non-ideal) voltage source is represented as an ideal voltage source in series with a resistance equal to its internal resistance Rj, aS shown in Fig. 3.20. This representation can be used to calculate the true terminal voltage of a source when current is drawn from it. Note that goa, internal resistance is an inherent property of Yoliage source; it is nota discrete component that can be Source measured with an ohmmeter. Only the external terminals are accessible to the user, so any and all measurements must be made at those terminals. ‘When load R, is connected across the terminals, the terminal voltage can be calculated using conventional analysis method. Q.11. What are the applications of current sources ? ‘Ans. Current sources have several applications : (i) They are used to represent certain devices that exhibit a constant-current behaviour. Transistors are examples of such devices. (ii) Current sources can also be used to represent equipment such as current-regulated power supplies. These supplies provide a constant current over some limited range of load resistance. (iii) They can be used in certain network analysis procedures. Q.12. What is the advantage of converting current source into voltage source and vice-versa ? ‘Ans. The reason one type of source is converted to other type often simplifies the analysis of a circuit containing both types. Any resistances that is in series with a voltage source, whether itbe internal or external resistance, can be included in its conversion to an equivalent current source. Similarly, any resistance in parallel with a current source can be included when it is converted to an equivalent voltage source. Let us illustrate this point with a numerical example. Referring to Fig. 3.21 (i), we want to find current in 6 kQ resistor by converting the current source to a voltage source. Since we want current in 6 kQ resistor, we use 3 kQ resistor to convert the current source to equivalent voltage source [See Fig. 3.21 (ii)]. 3k 5mA 15 mA 1s ma, oko ka Ska 62 45V oka @ ww (ay Fig. 3.21 The equivalent voltage source is shown in Fig. 3.21 (iit). E =1R=15mAx3kQ=45V 45V 45V C istor = — > Y= SV urrent through 6 Q resistor = 357 Erg = gq 7 MA Fig. 3.20 64 Objective Electrical Technology In the series circuit [See Fig. 3.21 (iif)], it would appear that current in 3 kQ resistor is also 5 mA. However, the 3 kQ resistor was involved in the source conversion, we cannot conclude that there is 5 mA in the 3 kQ resistor of the original circuit. Verify that current in the 3 KQ resistor in that circuit is, in fact, 10 mA. Q.13. Can you measure mesh currents ? Ans, Branch currents are the real currents because they actually flow in the branches and can be measured. However, mesh currents are fictitious and cannot be measured except in those instances where they happen to be identical with branch currents. Q.14. What is the basis of nodal analysis ? ‘Ans. Mesh analysis is based on Kirchhoffs’ voltage law. However nodal analysis is based on Kirchhoffs’ current law. Q.15S. What are the advantages of nodal analysis over Kirchhoffs' laws and mesh analysis ? Ans. The advantages of nodal analysis over Kirchhoffs laws and mesh analysis are : (0 Generallly less numebr of equations are required to solve a circuit by nodal analysis. (Nodal analysis is convenient in circuits where various voltages, referred to a common ground, are to be determined. (iii) Nodal analysis is a convenient approach to circuits that have current sources and conductances rather than voltage sources and resistances. Q.16, How is reference node chosen in nodal analysis ? Ans. The choice of reference node in nodal analysis is arbitrary but it is usually convenient to select the reference node as the one having the most components connected to it. Q.17. What are the advantages of Thevenin’s theorem ? Ans. Thevenin’s theorem offers the following advantages : (i Itreduces a complex circuit to a simple circuit viz a single source of emf. Ep, in series with a single resistance R,,. (ii) It greatly simplifies the protion of the circuit of the lesser importance and enables us to view the action of the output part directly. (iii) The theorem is particularly useful to find current in a particular branch of a network as the resistance of that branch is varied while ll other resistances and e.m.f. sources remain constant, Q.18. What is the difference between Thevenin’s theorem and Norton's theorem ? Ans, Although the Thevenin’s theorem and Norton’s theorem can be used to solve a given network, yet the circuit approach differs in the following respects : (i) Norton’s theorem is converse of Thevenin’s theorem in the respect that Norton equivalent circuit uses a current generator instead of voltage generator and the resistance Ry(which is the same as R,,) in parallel with the generator instead of being in series with it. (ii) Thevenin’s theorem is a voltage form of an equivalent circuit whereas Norton's theorem is a current form of an equivalent circuit. (iii) The mathematical relation between the two circuit forms i Ry=Rn, + Iy= Vind Rn Q.19. How will you convert Thevenin’s equivalent circuit into Norton's equivalent circuit and vice- versa ? Ans. (i) To convert Thevenin’s equivalent circuit into Norton’s equivalent circuit : Iy= Vr! Rm + Ry=Rry * (ii) To convert Norton's equivalent circuits to Thevenin’s equivalent circuit. Network Theorems 65 Q.20. Ans. Q.21. Ans. Q.22. Ans. Q.23. Ans. Q.24, Ans. Q.25. Ans. Vin=lyRy + Ry=Ry How does circuit behave under maximum power transfer conditions ? Under the conditions of maximum power transfer, the circuit exhibits the following characteristics : (i) The efficiency is 50% only as one-half of the total power generated is dissipated in the internal resistance of the source. (ii) The load voltage is one-half of the open-circuited voltage at the load terminals. What are two applications of maximum power transfer theorem ? ‘Two important applications of maximum power transfer theorem are given below : (i) In electronic circuits, maximum power transfer is usually desirable. For instance, in a public address system, the circuit is adjusted for maximum power transfer by making load resistance (i.e. speaker) equal to source (i.e. amplifier) resistance. (ii) Another example of maximum power transfer is found in the starting of a car engine. The power delivered to the starter motor of the car will depend upon the effective resistance of the motor and internal resistance of the battery. If the two resistance are equal (as is the case when the battery is fully charged), maximum power will be transferred to the motor to turn on the engine. This is particularly desirable in winter when every watt that can be extracted from the battery is needed by the starter motor to turn on the cold engine. If the battery is weak, its intemal resistance is high and the car does not start. How will you prove that efficiency under maximum power transfer conditions is 50% ? 2 Output power ___PR, = = Efficiency = “Tnput power ~ 72(R, +R) PR, 1 = a =4=50% - RLER, PxR2R, 2 (RR) Why is power system never operated under maximum power conditions ? Under the conditions of maximum power transfer, the efficiency is low (S0%) and there is, greater voltage drop in the lines. In a power system, the goal is higher efficiency rather than maximum power. For these reasons, maximum power transfer is not desired in a power system. Why are electronic ci its generally operated under maximum power transfer conditions ? In electronic circuits, it is often desirable to transfer maximum power e.g. (The signal power available at the receiving antenna is very small. It is very important to recover the maximum possible amount of signal power from the receiving antenna. (ii) In the public address system, it is desired that maximum power is transferred from the amplifier to the speaker (i.e. load) in order to operate the speaker. ‘To meet such situations in electronic circuits, we adjust the circuit for maximum power transfer. ‘The technique is to make the load resistance (e.g. speaker) equal to the source (e.g. amplifier) resistance. The circuit is then said to be matched. Where do we use star / delta and delta / star transformation ? There are some networks in which resistances are neither in series nor in parallel. A familiar example is a three terminals network e.g. delta network or star network. In such situations, it isnot possible to simplify the network by series and parallel circuit rules, However, converting delta network into star and vice-versa often simplifies the network and makes it possible to apply series-parallel circuit techniques. Objective Electrical Technology OBJECTIVE QUESTIONS 1. An active element in acircuit is one which () receives energy (ii) suppliesenergy (iii) both receives and supplies energy (iv) none of the above. 2. A passive element in acircuitis one which () suppliesenergy (ii) receives energy (iii) both supplies and receives energy (iv) none of the above 3. Anelectric circuit contains .. (active elements only (ii) passive elements only (iii), both active and passive elements (iv) none of the above 4. A linear circuit is one whose parameters (e.g. resistances etc.) (change with change in current (ii) change with change in voltage (iii)_do not change with voltage and current (iv) none of the above 5. Inthe circuit shown in Fig. 3.2 2, the number of nodes i . @ one (ii) two Gii) three (iv) four A A, 8 R, c Ry D Fig, 3.22 6. In the circuit shown in Fig. 3.22, there are junctions, Gi) three Gi) four (iii) two (iv) none of the above |. The circuit shown in Fig. 3.22, has.. branches. () two (ii) four (iii) three (iv) none of these 8. The circuit shown in Fig. 3.22 has..... loops. () two (i) four (iii) three iv) none of the above 9. In the circuit shown in Fig. 3.22, there are j . meshes. @ two (ii) three (iii) four (iv) five 10. To solve the circuit shown in Fig. 3.23 by Kirchhoff’s laws, we require . ii () one equation —_(ii) two equations (iii) three equations (iv) none of the above M1. To solve the circuit shown in Fig. 3.23 by nodal analysis, we require ..... () one equation (ii) two equations (iii) three equations (iv) noneof the above 32 49 Wi Wy ae Ie | 22 T4ov Fig. 3.23 12. To solve the circuit shown in Fig. 3.23 by superposition theorem, we require () onecircuit (ii) two cireuits (iii) three circuits (iv) noneof the above 13. To solve the circuit shown in Fig, 3.23 by Maxwell's mesh current method, we require ( oneequation (ii) three equations (iii) two equations (iv) none of the above 14. In the circuit shown in Fig. 3.24, the voltage at node B w.rt, D is calculated to be 15 V. ‘The current in 3 Q resistor will be .. (@ 2A Gi 5A ii) 2.5A (iv) none of the above 15. The current in 2 2 horizontal resistor in Fig. 3.24 is. (@ 10A (iii) 2A (i) 5A (i) 2.54 Network Theorems 39 e 22 AW WV ~ — sov 20 | D Fig, 3.24 16. In order to solve the circuit shown in Fig. 3.24 by nodal analysis, we require (ii) ovo equations iii) three equations (iv) none of the above 17. The superposition theorem is used when the cireuit contains (one equation @ aainglevollagezoarcs’ Gi) a number of voltage sources (iii) passive elements only (iv) none of the above 18. Fig. 3.25 (ii), shows Thevenin’s equivalent circuit of Fig. 3.25 (i). The value of Thevenin’s voltage Ey, is . ( 20V (i) 4N (ii) 12 (iv) 36 V 49 5Q 4 “a Fig. 3.25 19. ‘The value of Ry in Fig. 3.25 (ii) is @ 52 Gi) 352 (ii) 64.9 (iv) 714.2 67 20. The open-circuited voltage at terminals AB in Fig. 3.25 (iis... (@ RV iii) 24V (i) 20V (iv) 40V 21. Fortransfer of maximum powerin the circuit shown in Fig, 3.25 (i), the value of R, should be. () 352 (i) 64Q (iii) 749. (iv) 152 22. Fig. 3.26 (ii) shows Norton's equivalent circuit of Fig. 3.26 (i). The value of Ry is () 50 (i) 45.0 Gi) 1059 iv) none of the above 20 20 8 @ A Gi) Fig. 3.26 23. The value of /y in Fig. 3.26 (ii) is . (3A (i) 1A (iii) 2A (iv) none of the above Thevenin’s theorem is form of an equivalent circuit. (voltage (ii) current (iii) both voltage and current (iv) none of the above 25. Norton's theorem is .« Thevenin's theorem. (the same as (iii) none of the above (iv) cannot say (ii) converse of 26. In the analysis of a vacuum tube circuit, we generally use .. . theorem. (superposition (ii) Norton's (iii) Thevenin’s (iv) reciprocity 27. Norton's theorem is an equivalent circuit. (both voltage and current (i) current (iii) voltage (iv) none of the above 28. In the analysis of a transistor circuit, we usually use theorem. (® Norton’s (ii) Thevenin’s (iii) reciprocity _ (iv) superposition 29, Fig. 3.27 (i) shows Norton’s equivalent circuit of a network whereas Fig. 3.27 (ii) showsits Thevenin’s equivalent circuit, The form of value of Ey, is... - @ 15Vv (i) 0866 V (i) 3V (i) 6V Gi) Fig. 3.27 30. The value of Ry in Fig. 3.27 (i is @ 32a (22 (ii) 159 (o) 62 31. [fin Fig. 3.27 (i), the value of Jy is3 A, then value of E,, in Fig. 3.27 (ii) will be... " @ iV (iD 9V (i) SV (iv) none of the above 32. For transfer of maximum power, the relation between load resistance , and internal Objective Electrical Technology resistance R, of the voltage source is @ R,=2R, (i) R,=05R, (ii) RL=15R, (iv) RL=R, 33. Under the conditions of maximum power transfer, the efficiency is . () 75% (ii) 100 % (iif) 50% (iii) 25 % 34. The open-circuited voltage at the terminals of load R, is 30 V. Under the conditions of maximum power transfer, the load voltage will be. @® 30V (i) 10V (ii) SV (iv) SV 35. The maximum power transfer theorem is used in . (@ electronic circuits (ii) powersystem (iii) home lighting circuits (iv) none of the above 36. Under the conditions of maximum power transfer, a voltage source is delivering a power of 30 W to the load. The power generated by the source is .. 2 () 45W (ii) 30 W (ii) 6OW (iv) 90W 37. Forthecircuit shown in Fig. 3.28, the power transferred will be maximum when R, is equal to .... . ) 452 i) 62 (iii) 32 (iv) none of the above A Fig, 3.28 8 38. The open-circuited voltage at terminals AB in Fig. 3.28 i: ¥ @ Rv (ili) 15 (i) 95 V Network Theorems 39. If in Fig. 3.28, the value of R, = 6 Q, then current through R, is . @ 2A ( (iti) 1.75. A (iv) 1A 40. Under the conditions of maximum power transfer, the voltage across R, in Fig. 3.28 is. @® 6V (i) 4V (iii) 9V (iv) 12V 41. The output resistance of a voltage source is 4 Q. Its internal resistance will be @ 42 i) 22 (ii) 12 (iv) infinite 42. Deita/Star or star/delta transformation technique is applied to ... network. @ oneterminal (ii) two terminal (ii) three terminal (iv) none of the above 43. The resistor values in delta network that is, equivalent toa Wye containing three 120 2 resistors is ... - () 360Qeach (ii) 240 Qeach (iii) 180 Qeach (iv) 120 Qeach 44, The resistor values in Wye network that is equivalent to a delta containing three 12 resistors is, @ 2kQeach (ii) 4kQ each (li) 8 kQ each (iv) 6kQeach 45, When a load of 1 k@ is connected across a 20 mA current source, it is found that only 18 mA flows in the load. What is the internal resistance of the source ? 20 mA 20 mA oO Fig, 3.29 (i) 6kQ (i) 18k (i) 9kQ 46. What would be the terminal voltage of the current source (with R, connected) in Fig. 3.29 if the internal resistance of the source were 1.5kQ? @) 6V (i) 9V (iif) 12V (vy) 3V 47, The current in 3 kQ resistor in Fig. 3.30 by converting the current source into voltage source is smaCt) Fig. 3.30 @ 10mA (i) 12mA (iti) 6mA (iv) SmA 48, Using mesh analysis, the current in 4 Q resistor in Fig. 3.31 is 29 Fig. 3.31 (i) 3 Adownward (ii) 6 A upward (i) 9 Aupward (iv) 2 A downward 49, Using mesh analysis, the current in 6 Q resistor in Fig. 3,32 is 20 —— 16V BQ 18V Fig. 3.32 @ 032AL ii) 0.64 AT Gi) 0.32 AT (iv) O64 AL 50, A voltage source has a terminal voltage of 28 V whenits terminals are open-circuited. When a 12 Q load is connected across the terminals, the terminal voltage drops to 24 V. What is the internal resistance of the 70 source ? @ 052 qi) 1Q (i) 22 () 250 51. A 16 mA current source has an internal resistance of 10 kQ. How much current will flow in a 2.5 kQ load connected across its terminals ? @ 42ma (i) 6mA (i) Sma (@) 12.8mA 52. Convert the voltage source shown in Fig. 3.33 to an equivalent current source. 259 Fig. 3.33 (® 2-4A in parallel with 25 Qresistor (3A inparallel with 50 2 resistor (ii) 2-4 A series with 25 Qtesistor (iv) none of above. 53. In Fig. 3.33 above, what is the shorted terminal current in the equivalent current 6v source ? ( zero (i) 484 (ii) 24A () 124 54, Convert the current source shown in Fig. 3.34 to equivalent voltage source. 25 kn Fig. 3.34 (© 20V in series with 25 kQ (ii) 40 V in series with 25 kQ (if) 36V inparallel with 25 kQ (iv) none of above 55. By performing an appropriate source conversion, find the voltage across 120 Q. resistor in Fig. 3.35 402 60.9 ® Fig, 3.35 Objective Electrical Technology (9 40v (i) 30 (ii) 18V (iv) 20V 56. By performing an appropriate source conversion, find the voltage across 120 2 resistor in Fig. 3.36 62 WW aa zon (F)os5a Fig. 336 (@ 20V (i) 40V (ti) 30V (@) 60v 57. Convert the voltage source in Fig. 3.37 into an equivalent current source. - an 18 2K Fig. 337 () 4.5mA in parallel with 2kQ (i) 9 mA in parallel with? kQ (ii) 18 mA in series with2 kQ (i) none of the above 58. In the above question, what is the shorted terminal current in the equivalent current source ? () 4.5mA (i) 9mA (i) 12mA () 18mA 59. Using nodal analysis, what is the voltage at point 4 [See Fig. 3.38] ? A 0018 26 2aQ) @)2a Fig. 3.38 ( -15V (i) -50V (iii) +25V (iy) -10V Network Theorems 60. In Fig. 3.38, whatis the potential at point B by nodal analysis ? () SOV (i) -25V ii) - 50 (iv) -75V 61. Using superposition theorem, current in 10 2 resistor in Fig. 3.39 is 100 2 S130 Fig. 3.39 @ 200mA L Gi) about 500 mA L (i) 24mAT (iv) 208 ma T 62. What isthe power dissipated in 10 Qresistor in Fig. 3.39 above ? _@O5W (i) 75 W (ii) 45W (iv) 25 W 63, Find the Thevenin equivalent circuit to the left of terminals A and B in Fig. 3.40 (i) Ey=8VRy=4Q 64, Find the Thevenin equivalent circuit to the left of terminals x-y in Fig. 3.41 60 Q 100.2 x 24v) Fig. 3.41 @ E_=16V.R, = 1402 (ii) En, = 8 VRp,= 1202 18 V,R,,=722 71 (iv) none of above 65. Find Thevenin equivalent circuit lying tothe right of terminals x-y in Fig. 3.42 x 4kQ Fig. 3.42 @ IOV, Ry, = 1kQ w 15 V, Rp, = 10 kQ (ii) 12 V, Rp, =5kQ @) 66. Find the voltage across R, in Fig. 3.43 when R, = 1 kQ and 2 kQ. 1§kQ “sv on b A Fig. 3.43 @ 9VA2V (i) 4V.8V (iii) 16 V,24V (iv) 18 V, 36V 67. Find the Norton equivalent current source at terminals x-y in Fig. 3.44. th ; y —1By Fig. 3.44 @ 1y=4mA,Ry=5Q (i) y= 1A, Ry=3.52 (ii) 1p=2.5A,Ry=62 (iv) ly=3.3A,Ry= 6.672 68. Find Norton equivalent current to the left of terminals x-y in Fig. 3.45 800 8 x 200 2 200 2 Fig. 348 72 (i) y= 0.1 A (iii) Iy=0.01A (iv) none of above 69. The ammeter labelled A in Fig. 3.46 reads 35 mA. Is 2:2 kQ resistor shorted ? Assume ammeter has negligible resistance 25u 10mA\ Fig. 346 () No (ii) Yes (iii) may or may not be (iv) cannot say In Fig. 3.46, what should be the ammeter reading if 2.2 kQ resistor is not shorted ? (@) 1094mA (ii) 25 mA (iii) 1223mA (iv) G45 mA Find the value of R, in Fig. 3.47 to obtain maximum power in Ry. 300 2 25 2 100 V 100 2 3A 502 Fig, 3.47 () 1009 (ii) 752 Gii) 250.9 (iv) 1502 ‘72. In Fig. 3.47, find the maximum power in R,. @ 2W Gi) 1042 W Gi) 234W (iv) 452. W 73. What percent of the maximum power is Objective Electrical Technology (il) 68% of P, (max) (iii) 88-89% of P, (max) (iv) none of above 74, What percent of the maximum power is delivered to R, in Fig. 3.48 when R, = Ryf2? @) 65% (ii) 70 % (ii) 88:89 % (iv) none of above 75. Find Millman’s equivalent circuit w.r.t. terminals x-y in Fig. 3.49. Fig. 3.49 Single current source of 0-1 A and resistance 75 Q (ii) Single current source of 2 A and resistance 50 (iii) Single current source of 1 A and resistance 25 Q (iv) none of above 716. Use superposition principle to find current through R, in Fig. 3.50. Ry=1k2 8 mA Ry = Sk Fig. 3.50 ® Imac (i) 2MAK i) LSmA> (iv) 25 mA 77. Use superposition principle to find current through R, in the circuit shown in Fig. 3.51. Ry =602 “tT Fig. 3.51 Fig, 3.48 @ 02Ae (i) 025A (iii) 0.125A> (iv) O05 A> (i) 70% of P, (max) Network Theorems 78. Find Thevenin equivalent circuit to the left of terminals x-y in Fig. 3.52. 73 52 402 # 6a 102 102 @ (ii) BV 90 Q = Ry = ee 180. Q 12 Fig. 3.52 6 32 72 viky wm J, a /5.V:Rp,= 102 OV ; Ry =9Q ¥ : 79, Conver delta network shown in Fig. 3.53 80 Wht percentage ofthe maximum power i to equivalent Wye network times greater than the Thevenin resistance ane of the source to which it is connected ? @ 3% (ii) 40% (iil) 35% (iv) 33.06 % 30 20Q Fig. 3.53 ANSWERS TO OBJECTIVE QUESTIONS 1. (ii) 2. (ii) 3. (iid) 4. (iii) 5. (iv) 6. (iii) 7. iii) 8. (iii) % @ 10. (ii) iL @) 12. (ii) 13. (iii) 14. i) 15. (iv) 16. 17, (ii) 18. (ii) 19. (iv) 20. (iii) 21, (ii) 22. (it) 23, (iii) a (i) 25. (ii) 26. (iii) 27, (ii) 28. (i) 29. (iv) 30. (i) 31. (i) 32. (iv) 33. (iti) MM, (iv) 35. (i) 36. (iii) 37. (i) 38.) ». (iv) 40. (i) 41 42. (iii) 43. (0D 44. (ii) 45. (iv) 46. (iii) 47. 48. () 49. (ii) 50. (iii) Si. {iv) 52. (i) 53. (iii) 54. (ii) ‘55. (iv) ‘56. (iii) ‘S7. (ii) ‘S58. (if) 59% () 60. (ii) 61. (i) 62. (iv) 63. (iti) 64. (i) 65. (ii) 66. (i) 67. (iv) 68. (iii) 69. (ii) 70. (i) 71. (iv) 72. (ii) 73. (iti) 74. (iii) 75. {i) 76. (ii) 77. (iit) 78. {iv) 79. {i) 80. (iv) 74 Objective Electrical Technology HINTS TO SELECTED OBJECTIVE QUESTIONS 5. A node of a network is an equipotential surface at which nwo or more circuit elements are joined. Thus in Fig. 3.22, circuit elements R, and E, are joined at A and hence A is the node, Similarly B, Cand D are nodes, 6. A junction is that point in a network where three or more circuit elements are joined. In Fig. 3.22, there are only two junctions viz B and D. That B is a junction is clear from the fact that three circuit elements R,, R, and R, are joined atit. Similarly D is a junction because it joins three circuit elements R,, £, and E). 7. A branch is that part of a circuit which lies between two junction points. Thus referring to Fig. 3.22, there are a total of three branches viz BAD, BCD and BD. 8. A loopis any closed path of a network. Thus in Fig. 3.22, there are only three loops viz ABDA, BCDB and ABCDA. 9. A mesh is the most elementary form of a loop and cannot be further divided into other loops. In Fig. 3.22, both loops ABDA and BCDB qualify as meshes because they cannot be further divided into other loops. However, the loop ABCDA cannot be called a mesh because it encloses two loops ABDA and BCDB. 10. Since there are two unknown quantities (viz /, and J,), we require two equations. 12, Since there are two sources of voltage. wo circuits are required for analysis by superposition theorem. 14, Curent in 3 Q resistor = 22 = 1S wa 18. E,, = Open-circuited voltage at terminals AB = Current in62x6Q=4Ax6Q=24V 19. Ry, = Resistance at terminals AB with R, removed and battery replaced by a short. 4x6 S%8 452742 ar6t5 21. For transfer of maximum power, R, should be equal to output resistance at terminals AB. Now output resistance at terminals AB. =4Q|6Q2+52 = Ry, = 742. 22. Ry = Ry, = 2260432 : 2x6 = Fog +3=450 23. Iy = Current through terminals AB with load removed and terminals AB short-circuited. Load on source = 22+3Q/62 3x6 = 20+ 5°0 =40 Source current, / = 12/4=3A =3x-L= Wy = 3x 2p 52A 2% En, = 2Ax3Q=6V 30. Ry, = Ry=3Q 33. Suppose a source of voltage V and internal resistance R, is supplying a current of J to load R;, under maximum power transfer conditions. Network Theorems 7 Efficiency = [v Rp=R] 1 = $=50% 34. Load voltage under maximum power transfer, Vv =IR, (avR)® xR, = V2 =302=15V (Ce RL=R) -2R, i 36. Under maximum power transfer conditions, one-half of the total power generated is dissipated in the internal resistance R, of the source i-e., the efficiency is 50 %. Total power generated = 2x30 = 60W 41. The output resistance of source means its internal resistance. 45. Voltage across load. V, = 1,R, = 18 mA x 1kQ= 18 V Current in R,,,./,, = 20-18=2mA V8 Roa = 7 Dmd =O KO $0. When a load of 12 Q is connected across the terminals of the source, circuit current is ¥ 28 Is = RER, 12+ Ry Voltage drop across internal resistance = 28-24=4V 28 += (ig, Me or Ry, = 22 51. Fig. 3.54 shows the conditions of the problem. ‘The current source provides 16 mA to two parallel connected resistors. Therefore current in 2.5 k@ load is 10 A = xia age 16 x Me = 128 ma 16a 40.0. } 4 I 16 mat 10k2 25 kQ E=40V——- 120.2 60 2 Fig. 3.54 Fig. 3.55 55. Since we want to find voltage across 120 © resistor, we use 40 Q resistor to convert current source into equivalent voltage source. As shown in Fig. 3.55, = 1R=1Ax40Q2=40V Referring to Fig, 3.55, total circuit resistance is R, = 40+ 1BXED — 40 + 40 = 800 Circuit current,7 = £/R,= 40/80 =0.5 A 76 Objective Electrical Technology _ 0 60. 130+ 6 ~°°* 180 Voltage across 120.9 resistor = 1, x 120 = 0.1666 x 120= 20 V 61, Fig. 3.56 shows the solution of the problem 100 2 Current in 120 Q resistor,/, = 1x 77> = 0.1666 A 93.8 46.9 mA KL a ! “ ! (® Circuit with 13 V source removed 10099 Ba (iii) Original circuit with currents algebraically combined Fig. 3.56 64, Fig. 3.57 shows the solution of the problem. S02 100.2 sro ft Fn S02 Py = 1002+ 402 pith (24V)= 18 E, 120+ 60 Network Theorems 7 Ame 40a 1 Eqy=16V = 16v>— Q y Y (iii) (iv) Fig, 3.57 66. Fig. 3.58 shows the solution of the problem. 18k x Short SkQ S4— Ap, = 15 KQYIKA=1 KD (i) 154 45V Open = 3&2 = 3kO___(45V) = 30V "TSkQ +k y (ii) 15kQ x Short} 12mAl Ska E, = (4 mA) (3k = 12V vad ff y (ii) = Vo Be Gy) Fig. 3.58 vl 1k _}igs When R, = 1kQ5 Vy, (ata) 8-9" : vale When R, =2k2; V,=|TpQy7R | I8=12V 69. If the 2.2 kQ resistor is shorted, the ammeter current will equal the Norton equivalent current 78 73. 7S. Objective Electrical Technology of the circuit lying to the left of the resistor terminals. (We assume that the ammeter has negligible resistance.) InFig. 3.59 (ii), we calculate the portion of the Norton current due to the 10 mA current source acting alone. This value is clearly 10 mA. In Fig. 3.59 (iii), we open the current source to determine the contribution of the 25 V voltage source to ly. This current is 25 V/1 kQ = 25 mA. By the superposition principle fy = 10 mA +25 mA = 35 mA. (Zero resistance ammeter) 25V Short Brom tka gece 10mA JShort @ Gi 25v i}-—@) (A) |e mA 8 | | ee Tk Short ue os 25 mA 35mA SON Gi) (i) Fig. 3.59 Fig. 3.59 (iv) shows that a short connected across the terminals of the Norton equivalent circuit draws the entire Norton current of 35 mA through the ammeter. We conclude that the 2.2 kQ. resistor is shorted. As shown in Fig. 3.60, when R, = 2R,,,, the load voltage V, is found by voltage divider rule as under : Ani 2R, * Y= ae Em The power delivered to the load is then t= RR, Ry BE py Fig. 3.60 Since P,(max) = E’7y/4Ryy, the ratio of P, to P, (max) is PL _ SEAS Ry 16 Pimax) ~ “E2,/4Ry, 18 [i P, (max)| x 100 % = 88.89 % P,(max) Ew 2Ry V=$Ey, ‘Thus, . The resultant current of the three sources is x O2AT+OISAL+005AT =01AT The equivalent resistance of the three parallel 0-24 ~0.15.A resistors is +005A=0.1A 600 100] 00 600 QI} 169 SF" 6002 =752 Thus, the single equivalent current source has Fig 361” value 0.1 A and resistance 75 Q, as shown in Fig. 3.61. 4 Electrical Work, Power and Energy CHAPTER OVERVIEW 1, IMPORTANT PHYSICAL QUANTITIES It is profitable to give a brief description of the following physical quantities much used in science and engineering : () Mass. It is the quantity of matter possessed by a body. The SI unit of mass is kilogramme (kg). The mass of a body is a constant quantity and is independent of place and position of the body. Thus the mass of a body is the same whether it is on Earth’s surface, the Moon's surface, on the top of a mountain or down a deep well. Gi) Foree, Itis the product of mass (kg) and acceleration (ns"). The unit of force,is newton (N) : being the force required to accelerate a mass of 1kg through an acceleration of 1 avs? F = manewtons mass of the body in kg @ = acceleration in nvs? (iii) Weight, The force with which a body is attracted towards the centre of Earth is called the weight of the body. Now, force = mass x acceleration. If m is the mass of a body in kg and g is the acceleration due to gravity in m/s*, then, Weight, W = mg newtons As the value of g* varies from place to place on earth’s surface, therefore, the weight of the body varies accordingly. However, for practical purposes, we take g = 9-81 m/s" so that weight of the body =9.81m newtons. Thus if a mass of | kg rests on a table, the downward force oa the table Le., weight of the body is W = 9-81 x 1 = 9-81 newtons. ‘The following points may be noted carefully : (a) The mass of a body is a constant quantity whereas its weight depends upon the place or position of the body. However, it is reasonably accurate to express weight W = 9-81 m newtons where m is the mass of the body in kg. (6) Sometimes weightis given in kg. wt. units. One kg-wt means weight of mass of | kg ie..9-81 x 1=9-81 newtons. Ikgwt = 981 newtons ‘Thus when we say that a body hasa weight of 100 k, it means that it has a mass of 100kg and that it exerts a downward force of 100x 9-81 newtons. 2. UNIT OF WORK OR ENERGY Work is said to be done on a body when a force acts on it and the body moves through some distance. ‘This work done in stored is the body in the form of energy. Therefore, work and energy are measured in the same units, The SI unit of work or energy is joule or newton-meire and is defined as : where m " * The value of g is about 9°81 ns? at sea level whereas at equator, it is about 9-78 m/s? and at each pole it is about 9:832 mvs”. 79 80 Objective Electrical Technology The work done on a body is one joule if a force of one newton moves the body through I m in the direction of force. It may be noted that work done or energy possessed in an electrical circuit or mechanical system or thermal system is measured in the same units viz, joules. This is expected because mechanical, electrical and thermal energies are interchangeable. For example, when mechanical work is transferred into heat or heat into work, the quantity of work in joules is equal to the quantity of *heat in joules. 3. SOME CASES OF MECHANICAL WORK OR ENERGY It may be helpful to give a few important cases of work done or energy possessed in a mechanical system : (@® When a force of F newtons is exerted on a body and the body moves through a distance ‘d” metres in the direction of force, then, Work done = Fx djoules or Nm (i) Suppose a force of F newtons is maintained tangentially at a radius r metres from Oas shown in Fig. 4.1. In one revolution, the point of application of force travels through a distance of 2xr metres. Work done in one revolution = Force x Distance moved in | F revolution Fig. 41 = Fx2nr 22x TjoulesorNm where T= Fr is the torque. Clearly, the SI unit of torque will be joules or Nm. If the body makes N revolutions per minute, then, Work done/minute = 2 NT joules (iif) Ifa body of mass m kgis moving with a speed of v m/s, then kinetic energy possessed by the body is given by ; K.E. of the body = bay ‘joules w) Ifa body having a mass of m kg is lifted vertically through a height of h metres and g is acceleration due to gravity in m/s’, then, Potential energy of body = Work done is lifting the body Force required x height Weight of body x height = mgxh=mghjoules. 4. ELECTRICAL ENERGY The SI unit of electrical work done or electrical energy expended ina circuit is also joule —exactly the same as for mechanical energy. It is defined as under : One joule of energy is expended electrically when one , Q=I R B coulomb is moved through a p.d. of 1 volt. nw ‘Suppose a charge of Q coulomb moves through a p.d. of V volts as shown in Fig. 4.2. Then electrical energy expended if ———— ys Fig 4.2 * Although heat energy was assigned a separate unit viz, calorie but the reader remembers that | calorie = 4-186 joules. In fact, the thermal unit caloric is obsolete and now-a-days heat is expressed in joules. Electrical Work, Power and Energy 81 Electrical energy expended = VQ joules Vit joules P Rtjoules we joules Itmay be mentioned here that joule is also known as watt-second i.e. 1 joule = 1 wat we are dealing with large amount of electrical energy, it is often convenient to express it in kilowatt hours (k Wh). 1k Wh = 1000 watt-hours 1000 x 3600 watt-sec or joules 1k Wh = 36x 10° joules or watt-sec Although practical unit of electrical energy is kWh, yet it is easy to see that this unit convertible to joules with the help of above relation. 5. THERMAL ENERGY The thermal energy was originally assigned the unit ‘calorie’, One calorie is the amount of heat required to raise the temperature of 1 gm of water through 1°C. If Sis the specific heat of a body, then amount of heat required to raise the temperature of m gm of body through 0°C is given by : Heat gained = m SO calories. Ithas been found experimentally that 1 calorie = 4-186 joules so that heat energy in calories can be expressed in joules as under : Heat gained = (m S@) x 4-186 Joules The reader may note that SI unit of heat is also joule. In fact, the thermal unit calorie is obsolete and unit joule is preferred these days. 6. UNITS OF POWER The rate of doing work is called power. In other words, power is the work done per unit time. Since work is measured in joules and time in seconds, the unit of power will be joule /sec or watt. (i)_In practice, watt is often found to be inconveniently small, consequently the unit kilowatt (kW) is used. One kW is equal to 1000 watts i.e. 1kW = 1000 watts For larger powers, the unit megawatt (MW) is used. One megawatt is equal 10 1000 kW i.e. 1MW 1000 kW = 1000 x 1000 watts * 1MW = 10° watts (ii) Sometimes power is measured in *horse power (H.P.}. LHP = 746 wats (ii) If a body makes N rp.m. and the torque acting is T newton-metre, then, 2n NT Work done / second = J/s or watts * This unit for power was conceived by James Watt, a Scottish scientist who invented the steam engine. In his experiments, he compared the output of his engine with the power a horse could put out. He found that an “average” horse could do work at the rate of 746 joules/sec. Although power can be expressed in watts or kW, the unit H.P. is still used. 82 Objective Electrical Technology 2 te. Power = ae watts Since 1H.P. = 746 watts _ 2" NT Power = Giaag HP where Tis in newton-metre and Nis in p.m. 7. HEATING EFFECT OF ELECTRIC CURRENT When electric current (i.e. flow of free electrons) passes through a conductor, there is a considerable ‘friction’ between the moving electrons and the molecules of the conductor. The electrical energy supplied to the conductor to overcome this ‘electrical friction’ (which we refer to as electrical resistance) is converted into heat. This is known as heating effect of electric current. For example, if amperes is flowing through a conductor of resistance R ohms for t seconds, the electrical energy supplied is I° Ri joules, This energy is not destroyed but the whole of /° Rr joules is changed into heat. The heating effect of electric current is utilised in the manufacture of many heating appliances such as electric heater, electric toaster, electric kettle, soldering iron etc. The basic principle of all these appliances is the same. Electric current is passed through a high resistance (called heating element), thus producing the required heat. There are a number of substances used for making a heating element. One that is commonly used is an alloy of nickel and chromium, called nichrome, This alloy has a resistance more than 50 times that of copper. The heating element may be either nichrome wire or ribbon wound on some insulating material that is able to withstand heat. While dealing with problems on heating appliances, the following points may be kept in mind : (i) The electrical energy ink Wh can be converted into joules by the following relation : 1k Wh = 36x 10° joutes (ii) The heat energy in calories can be converted into joules by the following relation I calorie = 4-186 joules kcal = 4186 joules (iii) The electrical energy in kWh can be converted into calories (or kilocalories) by the following relation : 1kWh = 36x 10°joules _ 36x 10° ~ S186 = 860x 10° calories a 1k Wh = 860k cal (iv) The electrical energy supplied to the heating appliance forms the input energy. The heat obtained from the device is the output energy. The difference between the two, if any, represents the loss of energy during conversion from electrical to heat energy. 8. FUSE A fuse is a short piece of metal, inserted in the circuit, which melts when excessive current flows through it and thus breaks the circuit. Fuse A fuse is generally made of materials having low melting pointand high conductivity. Itis insertedin series with the circuit to be protected and carries the total circuit current as shown in Fig. 4.3. Under normal conditions, the fuse element is at a Fig. 4.3 calories Toaster Heater leh+heh Electrical Work, Power and Energy 83 temperature below its melting point. Therefore, it carries the normal current without overheating. However, when a short-circuit or overload occurs, the current through the fuse increases beyond its normal value. This raises the temperature and fuse element melts (or blows out), disconnecting the circuit protected by it. In this way fuse protects the electrical appliances from damage due to excessive currents. ‘Note : When a fuse blows, it should be replaced by fuse wire of the same thickness. On no account should it be replaced by a fuse wire of greater thickness, 9, POWER DISSIPATION Power dissipation usually refers to the rate at which electric energy is converted into heat. A resistor in an active circuit may feel warm to touch. This is due to the conversion of electric energy into heat energy. The resistor dissipates the heat energy by transferring it to surrounding structure and to the atmosphere. A resistor that is dissipating a large power may become hot enough to burn a finger. If the power dissipation is high, there will be a rapid conversion of electric energy into heat energy. As a result, the resistor reaches an excessive temperature and is damaged. Resistors and other equipment have maximum power dissipation ratings. For example, a resistor may have a power dissipation rating of 5 W. It means that maximum current through the resistor should be such that power developed (P= R) does not exceed 5 W otherwise the resistor will be damaged due to excessive heat. Illustration. Let us illustrate the power dissipation of a resistor (or any other electrical device) with numerical examples. (i) A0.1.Q resistor is rated at 5 W. Is this resistor safe when conducting a current of 10.A ? P=PR = (10) x01=10W ‘The resistor is not safe since the calculated power exceeds the dissipation rating. (ii) What is the maximum safe current flow ina 47Q, 2-Wresistor ? =IR = j= /2 <0. a 2 fE-fE-o210 Ifthe current through this resistor exceeds 0-21 A, the resistor is likely to be damaged due to excessive heat (iii) What is the maximum voltage that can be applied across a 100 Q, 10 - W resistor in order 10 keep within resistor's power rating ? 2 Pode R or V = JPXR = J10x100 =31.6V If the voltage across this resistor exceeds 31-6 V, the resistor is likely to be damaged due to excessive heat. 10. EFFICIENCY Non-useful output (heat) The efficiency of a device or of a circuit is the ratio of useful output to the input ie. Input The law of conservation of energy states ‘that Useful ga “energy cannot be created or destroyed but can output be converted from one form to another”. Some (work) of the energy in electric circuits may be = Input from converted into a form that is not useful. For energy source example, consider an electric motor shown in Fig. 44 aa You have either reached a page that is unavailable for viewing or reached your viewing limit for this book. aa You have either reached a page that is unavailable for viewing or reached your viewing limit for this book. Ans, Qi. Qs. Q16. Ans, . The greater the rate of dissipation of energy (i Objective Electrical Technology . A 100 W bulb and 60 W bulb both rated at 220 V are connected in series across 220 V supply. 1f60 W bulb is removed and only 100 W bulb remains in the circuit, will illumination increase or decrease ? power), the greater is the illumination, p, = FB 10x60 _ TP +P, 100+60 For the second. P; = 100 W. Hence, the illumination will increase. For the first ¢: 7-5 W . A toaster produces more heat as compared to an electric bulb when these are connected in parallel. Which of the two has larger resistance ? Since the two are connected in parallel, the rate of production of heat is P= V/R. As the toaster produces more heat, its resistance must be less 1a 1000 W than that of the bulb (- Vis the same in the two cases). Heater Hence, electric bulb has larger resistance. A heater is designed to operate with a power of 1000 W on 100 Vine. It is connected to two resistances of hy 52 10 Q and 5 Q as shown in Fig. 4.5. What is the power output of the heater ? Resistance of the heater = V°/P = (100)'/1000 = 10 Q. + oe The equivalent resistance of parallel circuit is Fig. 45 — 10x5_50_10 "iors" is 3° ‘Total circuit resistance, Ry = 10 + 10/3 = 40/3 Q Circuit current, / = >-= Current in the heater = Power output of the heater = (2'5)* x 10= 62-5 W The wires supplying current to an electric bulb are not heated whereas the filament of the bulb becomes white hot. Explain. . The resistance of the wires per unit length is negligible as compared to that of the filament of the bulb. As a result, the rate of production of heat in the wires is so small to cause any noticeable rise of temperature. On the other hand, the rate of heat production in the bulb is very high. Consequently, the filament becomes white hot. Two electric lamps are rated for 60 W, 220 V and 40 W. 220 V. Find the heat generated in each lamp per second when they are connected in series across 220 V. 2 2 Vv? _ (220) _ 2420 si = VEY 240 9 Resistance of 60 W lamp, Ry = “p= Fy a ¥? _ (220) 2 = OR dO When the lamps are joined in series, total resistance Ry is Ry = R,+R,= 7204+ 1210- OQ =1210Q Resistance of 40 W lamp, Current in each lamp, 7 = ViR,= 220 x 3/6050 = 6/55 A Heat produced in 60 W lamp per second aa You have either reached a page that is unavailable for viewing or reached your viewing limit for this book. aa You have either reached a page that is unavailable for viewing or reached your viewing limit for this book. aa You have either reached a page that is unavailable for viewing or reached your viewing limit for this book. disconnected from the circuit by opening switch S, then incandescence of bulb B, will B, Mains Fig. 4.7 (id) increase (iv) cannot say () decrease (iii) No change 18. You are given the following electrical appliances (a) 1 kW, 250 V electric heater (b)_ 1 kW, 250 V, electric kettle (c) 1 KW, 250 V, electric bulb Which of these has the highest resistance ? (@ heater (i) kettle (iii) bulb (iv) all have equal resistances 16. If current in an electric bulb drops by 2%, then power decreases by @ 2% i) 4% Gi) 1% Gi) 16% 17. Two bulbs of $00 W and 300 W are manufactured to operate on 220 V line. The ratio of resistance of 500 W bulb to that of 300 W bulb is @ $:3 (i) 9:25 (ii) 3:5 (iv) 25:9 18. ‘The same mass of copper is drawn into two wires Imm thick and3 mm thick. Two wires are connected in series and current is passed. Heat produced in the wites is in the ratio @ 8h: (i) 1:81 (i) 321 (iv) 9:1 19. ‘The power of a heater is 500 W at 800°C. What will be its power at 200°C 2 Temperature coefficient of resistance is 4x 107°C, @ 484w (i) 672. W (ii) $26 W () 620 W ‘A constant voltage is applied between two Objective Electrical Technology ends of a uniform metallic wire, Some heat is developed. If both length and radius of wire are halved, the heat developed in the same duration is (half (ii) twice (iii) one-fourth (iv) same 21. The heat produced in a conductor of resistance 4-2. with 10 A flowing in it is () 42calisec (di) 100 calfsec (ili) 42cavsec (iv) 420 cal/sec 22, A heater coil rated at 1000 W, 220 V is con- nected to 110 V line, Power consumed is (®) 250W (i) 500 W (ii) 200 W (i) 400 W 23, Two electric bulbs marked 500 W, 220 V are put in series with 110 V line. The power dissipated in each bulb is ) 25 25 @ a ) % w Gi) PW ww > w 24, An clectric fan and a heater are marked as 100 W, 220 V and 1000 W, 220 V respectively. The resistance of heater is (@ zero (ii) greater than that of fan (iii) less than that of fan (iv) equal to that of fan 25. Two resistances connected in parallel across a.cell of negligible internal resistance, use 4 times the power that they would use, when in series across the same cell. If one of the resistances is 10 Q, the resistance of the other is, (i) 20Q (ii) 10Q (it) 402 (iv) 5Q 26. In the circuit shown in Fig. 4.8, the heat produced in 5 Q resistors 10 cal /sec. The heat produced in 4 Q resistor will be 4Q 62 >— WW “WW-

You might also like